39
BYJU’S IAS PRELIMS TEST SERIES 2021

BYJU’S IAS PRELIMS TEST SERIES 2021 · 2021. 1. 29. · 2. Answer: (c) Explanation: Recently, a commemorative declaration marking the 75th anniversary of the signing of the Charter

  • Upload
    others

  • View
    3

  • Download
    0

Embed Size (px)

Citation preview

Page 1: BYJU’S IAS PRELIMS TEST SERIES 2021 · 2021. 1. 29. · 2. Answer: (c) Explanation: Recently, a commemorative declaration marking the 75th anniversary of the signing of the Charter

BYJU’S IAS PRELIMS TEST SERIES

2021

Page 2: BYJU’S IAS PRELIMS TEST SERIES 2021 · 2021. 1. 29. · 2. Answer: (c) Explanation: Recently, a commemorative declaration marking the 75th anniversary of the signing of the Charter

`

1 | SEC-ECO-01

ALL INDIA PRELIMS TEST SERIES – 2021

TEST CODE: SEC-ECO-01

ANSWER KEY

1. Answer: (c)

Explanation: Net National Product at factor cost is also called national income. Net National Product at factor cost is equal to the sum of value added at factor cost or net domestic product at factor cost and net factor income from abroad. Net national product (NNP) is the monetary value of finished goods and services produced by a country's citizens, overseas and domestically, in a given period. Net National Product (NNP) = Gross National Product - depreciation + indirect taxes + subsidies. Statement 1 is correct: SInce the question talks about 'Net' National Product, depreciation has to be deducted from the Gross National Product. Therefore, NNP doesn't include depreciation. Statement 2 is incorrect: Since NNP measures the monetary value of finished goods and services produced by a country's citizens irrespective of the place of work, the income of Indian ambassadors working abroad forms a part of NNP. Statement 3 is correct: Income of a US-based MNC operating in Bangalore will be counted as income in NNP of the USA. Statement 4 is correct: As NNP excludes indirect taxes, taxes on petrol and diesel are excluded from its calculation. Statement 5 is correct: As NNP excludes subsidies, food subsidy is excluded from its calculation.

2. Answer: (c)

Explanation: Recently, a commemorative declaration marking the 75th anniversary of the signing of the Charter of the United Nations (UN) was delayed as no agreement on phraseology could be reached by the Member States. The Five Eyes refers to a group of five countries namely, Australia, Canada, New Zealand, the United Kingdom and the United States. The Five Eyes (FVEY) network is an intelligence-sharing alliance between these five countries. Hence, Japan and India are not a part of the Five Eyes network. Extra information: Recently these five countries and India objected to the use of a phrase “shared vision of a common future”, which is often associated with China. The U.K. ambassador to the UN broke the “silence process'' on behalf of six countries. The ambassador sent a letter on behalf of the six countries to the President of the 74th General Assembly, suggesting alternative wording. “Silence process” is a procedure by which a resolution gets passed if no formal objections are raised within a stipulated time. However, China, on behalf of itself and Russia, Syria and Pakistan raised objections to the silence being broken. Given the impasse, the UN General Assembly President has suggested an alternatively phrased declaration, which he has placed under the silence procedure.

Page 3: BYJU’S IAS PRELIMS TEST SERIES 2021 · 2021. 1. 29. · 2. Answer: (c) Explanation: Recently, a commemorative declaration marking the 75th anniversary of the signing of the Charter

`

2 | SEC-ECO-01

3. Answer: (a)

Explanation: Statement 1 is correct: The GDP growth rate (annual) has been declining since 2016-17 according to the Ministry of Statistics and Program Implementation. Although, some quarters have shown increase in the GDP growth rate but the cumulative annual GDP growth rate has declined. Even before the CoVID-19 pandemic, the 4th quarter of fiscal year 2019-2020 showed a 3.1% growth rate.

Figure: GDP figures in recent past

Source: Indian Express

Statement 2 is incorrect: India’s economic growth slipped to -23.9% in the first quarter (April-June) of 2020-21. It is the first economic contraction since 1980, and the maiden quarterly GDP fall on record, following limited activity during the June quarter amid lockdowns to control the spread of the coronavirus pandemic. Statement 3 is incorrect: Foreign financial institution grants to Indian government will lead to expenditure by the government and that contributes to an increase in demand in the market. This will result in higher production levels. So, indirectly foreign grants do contribute to GDP growth.

4. Answer: (b)

Explanation: The total stock of money in circulation among the public at a particular point of time is called money supply. RBI publishes figures for five alternative measures of money supply, viz. M0, M1, M2, M3 and M4. They are defined as follows:

M0 = Currency in circulation + Bankers’ deposits with the RBI + ‘Other’ deposits with the RBI

M1 = CU (currency held by the public) + DD ( Net demand deposits held by commercial banks)

M2 = M1 + Savings deposits with Post Office savings banks

M3 = M1 + Net time deposits of commercial banks

M4 = M3 + Total deposits with Post Office savings organisations (excluding National Savings Certificates)

Statement 1 is incorrect: M2 is equal to the sum of M1 and savings deposits with Post office savings banks. Statement 2 is correct: M3 is the most commonly used measure of the money supply. It is also known as a broad monetary aggregate as it includes all physical currency circulating in the economy (banknotes and coins), operational deposits in the central bank, money in current accounts, savings accounts, money market deposits, certificates of deposit, all other deposits and repurchase agreements. Statement 3 is incorrect: M1 and M2 are known as narrow money. M3 and M4 are known as broad money. These graduations are in decreasing order of liquidity. M1 is the most liquid and easiest for transactions whereas M4 is the least liquid of all.

Page 4: BYJU’S IAS PRELIMS TEST SERIES 2021 · 2021. 1. 29. · 2. Answer: (c) Explanation: Recently, a commemorative declaration marking the 75th anniversary of the signing of the Charter

`

3 | SEC-ECO-01

5. Answer: (d)

Explanation: In June 2020, the Government launched a pan-India Real-Time Market in electricity. The real-time market is an organised market platform enabling buyers and sellers to meet their energy requirements closer to real-time operation. The purchased electricity will be routed by the Power System Operation Corporation Limited (POSOCO) from supply sources to consumption point with help of Regional Load Despatch Centres. Statement 1 is correct: Real-time market is an organised market platform enabling buyers and sellers to meet their energy requirement closer to real-time operation. Under this, auctions will be held 48 times

a day, once every half an hour. To implement the scheme amendments were made to Power Market regulations, Indian Electricity Grid Code (IEGC) Regulations, and Open Access in inter-state transmission regulations. Statement 2 is correct: As of now, it is operational on two platforms, Indian Energy Exchange (IEX) and Power Exchange India Limited (PXIL). Indian Energy Exchange Limited (IEX) is the first energy exchange in India, providing an automated trading platform for delivery of electricity, Renewable Energy Certificates and Energy Saving Certificates. Power Exchange India Limited (PXIL) is India’s first institutionally promoted Power exchange providing an electronic platform for transactions in power and allied products. Statement 3 is correct: Presently, renewable energy is volatile in its generation due to its dependence on seasonal and weather factors, for example, solar energy is not available after sunset. Thus, it becomes a challenge for the

stabilization of the grid system. This real-time market helps to integrate higher quantum of renewable energy resources into the grid. Therefore, it would help India to achieve its renewable energy targets under the Paris Climate Deal.

6. Answer: (a)

Explanation: The Regional Rural Banks were set up on the basis of the Narasimham Working Group recommendations (1975). The aim is to develop the rural economy by providing credit and other facilities, in particular to small and marginal farmers, farmworkers, craftsmen and small entrepreneurs, to develop agriculture, commerce, trade, industry and other productive activities in rural areas, and for other related matters. Statement 1 is correct: On October 2, 1975, the first Regional Rural Bank, the Prathama Grameen Bank, was founded. Statement 2 is correct: As per RBI guidelines, the RRBs have to provide 75% of their total credit under PSL (Priority Sector Lending). Statement 3 is incorrect: The equity of a regional rural bank is owned in a ratio of 50:15:35 by the central government, the state government and the sponsor bank respectively.

7. Answer: (c)

Explanation: Depreciation is an accounting method of allocating the cost of a tangible or physical asset over its useful life or life expectancy. The depreciable amount of an asset is the cost of an asset or other amount substituted for cost, less its residual value. The useful life of an asset is the period over which an asset is expected to be available for use by an entity, or the number of production or similar units expected to be obtained from the asset by the entity. Statement 1 is correct: Depreciation is calculated by dividing the cost of the goods by the number of years of its useful life. Statement 2 is incorrect: Depreciation rate is calculated for fixed assets like buildings, furniture, office equipment, machinery,

Page 5: BYJU’S IAS PRELIMS TEST SERIES 2021 · 2021. 1. 29. · 2. Answer: (c) Explanation: Recently, a commemorative declaration marking the 75th anniversary of the signing of the Charter

`

4 | SEC-ECO-01

etc. But land is the only exception which cannot be depreciated as the value of land appreciates with time. Statment 3 is correct: The depreciation rates are notified by the Central Board of Direct Taxes (CBDT) under the Ministry of Finance yearly.

8. Answer: (c)

Explanation: Statements 1 and 2 are incorrect: NITI Aayog has constituted the ‘Himalayan State Regional Council’ to ensure sustainable development of the Indian Himalayan region. It is not chaired by the Union Defence Minister. The Council is chaired by the Prime Minister or the person nominated by him. The first chairman of this council was Dr V. K. Saraswat, Member, NITI Aayog. Statement 3 is incorrect: The members of the council consist of the Chief Secretaries of the Himalayan States as well as the Secretaries of key Central Ministries, senior officers of NITI Aayog as well as special invitees. The Himalayan States Regional Council will be the nodal agency for Sustainable development in the Himalayan Region. Statement 4 is correct: The council consists of the two union territories namely, Ladakh and Jammu & Kashmir, and eleven states namely Uttarakhand, Himachal Pradesh, Arunachal Pradesh, Manipur, Meghalaya, Mizoram, Nagaland, Sikkim, Tripura, two districts of Assam namely Dima Hasao and Karbi Anglong and Darjeeling and Kalimpong in West Bengal.

9. Answer: (b)

Explanation: In June 2020, National Commodity and Derivatives Exchange (NCDEX) launched the country’s first agriculture futures index called AGRIDEX.

Futures are a type of derivative instrument. A derivative is an instrument whose value is derived

from the value of one or more underlying assets, which can be commodities, precious metals, currency, bonds, stocks, stocks indices, etc.

In futures, there is an agreement to

buy or sell a specified quantity of financial instrument or physical commodity in a designated future

month at a price agreed upon by the buyer and seller.

Statement 1 is incorrect: It was launched by the National Commodity and Derivatives Exchange (NCDEX). It is the country’s leading agricultural commodity exchange, which offers services across the entire value-chain of agricultural commodities. It brings buyers and sellers together through its electronic trading platform. NCDEX has partnered with National Stock Exchange (NSE) Indices, a leading Index service provider, to maintain and disseminate real-time NCDEX AGRIDEX values. Statement 2 is correct: AGRIDEX is India’s first return based agricultural futures Index which tracks the performance of the ten liquid commodities (both Kharif and Rabi seasons) traded on the NCDEX platform.

The ten liquid commodities are Castor seed, Chana, Coriander, Cotton Seed Oil cake, Guar Gum, Guar Seed, Jeera, Mustard Seed, Refined Soya oil and Soybean. To ensure diversification in the index, no group of related commodities may constitute more than 40% of the total weightage. It will facilitate the participants in hedging their commodity risk based on price anticipation of the products.

Statement 3 is correct: The index can be used as an effective hedging tool in the futures market and can be used for diversification and risk management effectively. Hedging is a risk management strategy employed to offset losses in investments by taking an opposite position in a related asset.

Page 6: BYJU’S IAS PRELIMS TEST SERIES 2021 · 2021. 1. 29. · 2. Answer: (c) Explanation: Recently, a commemorative declaration marking the 75th anniversary of the signing of the Charter

`

5 | SEC-ECO-01

10. Answer: (c)

Explanation: PPIs are instruments that facilitate purchase of goods and services, remittances, funds transfers, etc., against the value stored in such instruments. Example: Oxigen, MobiKwik, etc. Statement 1 is correct: PPI issuers are companies incorporated in India and registered under the Companies Act, 1956 (amended in 2013). They participate in a payment system for issuing PPIs to individuals/organisations. A company can issue and operate PPIs after receiving authorisation from RBI. Statement 2 is incorrect: No interest is payable on PPI balances. Statement 3 is correct: These payment instruments are licensed and regulated by the Reserve Bank of India under the Payment and Settlement Systems (PSS) Act, 2007.

11. Answer: (b)

Explanation: The Reserve Bank of India has made it mandatory for all banks to connect all new floating rate loans (i.e. personal/retail loans, MSME loans) to an external benchmark with effect from 1 October 2019. The goal of the change is to speed up the transmission of monetary policy rates to customers.

Statement 1 is incorrect: Banks can select from one of four external benchmarks—repo rate, three-month Treasury bill yield, six-month Treasury bill yield, or any other benchmark interest rate reported by Financial Benchmarks India Private Ltd. The interest rate under the external benchmark shall be reset at least every three months. Statement 2 is correct: Subsequent to the introduction of an external benchmark system, the loans and credit limits linked to the MCLR, the base rate or the Benchmark Prime Lending Rate,

continued until repayment or renewal. Customers wishing to move to the repo-linked rate may do so on mutually acceptable terms.

12. Answer: (b)

Explanation: Pair 1 is incorrectly matched: When inflation is in single digit numbers ranging from 4% to 10%, it is called walking inflation. At this rate, inflation is not a major problem, but when it rises over 4%, Central Banks will be increasingly concerned. Pair 2 is incorrectly matched: Creeping or mild inflation is when prices rise 3% a year or less. According to the Federal Reserve, when prices increase by 2% or less, it benefits economic growth. Pair 3 is correctly matched: Galloping inflation is an inflation rate ranging from 20% to 1000%. At this rapid rate of price increases, inflation is a serious problem and it will be a challenge to bring it under control. Pair 4 is correctly matched: A hyperinflation is an extreme form of inflation – usually over 1,000% though there is no specific definition. Hyperinflation usually involves prices changing so fast, that it becomes a daily occurrence, and under hyperinflation, the value of money will rapidly decline.

13. Answer: (d)

Explanation: India plans to build a global ecosystem of interconnected renewable energy resources through the OSOWOG initiative that will be seamlessly shared for mutual benefits and global sustainability. The vision behind the OSOWOG is “The Sun

Never Sets'' and is a constant at any given point of time at some geographical location, worldwide. Solar energy can therefore be used through the interconnected transmission. The ISA may also be leveraged by the global grid plan.

Page 7: BYJU’S IAS PRELIMS TEST SERIES 2021 · 2021. 1. 29. · 2. Answer: (c) Explanation: Recently, a commemorative declaration marking the 75th anniversary of the signing of the Charter

`

6 | SEC-ECO-01

Statement 1 is correct: Recently, the Ministry of New and Renewable Energy (MNRE) issued a request for proposal

(RFP) for developing a long-term vision, implementation plan, road map, and institutional framework for its One Sun One World One Grid (OSOWOG) program. Statement 2 is correct: The blueprint for the OSOWOG will be developed under the World Bank’s technical assistance programme that is implemented to accelerate the deployment of grid-connected rooftop solar installations. Statement 3 is correct: International Solar Alliance (ISA) will be the implementing agency for carrying out all the activities of OSOWOG initiative. ISA will handle the bid process management and all subsequent activities of the initiative. This would also help in creating regional and international interconnected green grids which can enable sharing of renewable energy across international borders.

Extra Information: The interconnected grid is envisioned with India at the fulcrum and two broad zones viz.

The Far East which would include countries like Myanmar, Vietnam, Thailand, Laos, Cambodia etc., and,

The Far West which would cover the Middle East and the African Region.

14. Answer: (a)

Explanation: The concept of marginal utility is used by economists to determine how much of an item consumers are willing to purchase. Statement 1 is correct: Marginal utility is the added satisfaction that a consumer gets from having one more unit of a good or service. Statement 2 is incorrect: Marginal utility can be positive, negative or even zero. Positive marginal utility occurs when the consumption of an additional item increases the total utility. On the other hand, negative marginal utility occurs

when the consumption of one more unit decreases the overall utility. Although marginal utility tends to decrease with consumption, it may or may not ever reach zero depending on the goods consumed. Statement 3 is incorrect: The law of diminishing marginal utility is often used to justify progressive taxes. The imposition of higher taxes causes less loss of utility for someone with a higher income, thus everyone gets diminishing marginal utility from money.

15. Answer: (b)

Explanation: Recently, India was elected as a non-permanent member of the United Nations Security Council with an overwhelming majority. Statement 1 is incorrect: The UNSC consists of 15 members — five permanent members and 10 non-permanent members. The five permanent members are the US, UK, Russia, China and France. Every year, five non-permanent members are elected for a tenure of two years. To be elected as a non-permanent member of the council, each member-country requires a two-third

majority of the entire general assembly. Statement 2 is correct: Every non-permanent member is elected for a two-year term. The current two-year term will begin on 1 January 2021, and along with India — Ireland, Mexico and Norway were also elected to the council. Statement 3 is incorrect: India was elected as a non-permanent member of the United Nations Security Council with an overwhelming majority. This is the eighth time India has been elected a non-permanent member of the UNSC. Previously, it has been elected as a member for the period of 1950-1951, 1967-1968, 1972-1973, 1977-1978, 1984-1985, 1991-1992 and 2011-2012.

Page 8: BYJU’S IAS PRELIMS TEST SERIES 2021 · 2021. 1. 29. · 2. Answer: (c) Explanation: Recently, a commemorative declaration marking the 75th anniversary of the signing of the Charter

`

7 | SEC-ECO-01

16. Answer: (a)

Explanation: Statement 1 is correct: The Gross Domestic Product (GDP) deflator is a measure of general price inflation. It is calculated by dividing the nominal GDP by the real GDP and then multiplying it by 100. Statement 2 is incorrect: The GDP Deflator is a much broader and comprehensive measure. GDP Deflator reflects the prices of all domestically produced goods and services in the economy whereas, CPI is based on a limited basket of goods and services, and WPI is based on a limited basket of goods only. Statement 3 is incorrect: The GDP Deflator includes the price of investment goods, government services and exports, but excludes the price of imports.

17. Answer: (c)

Explanation: Statement 1 is incorrect: The NRE account or Non-Resident (External) Rupee Accounts can be opened by NRIs, PIO (Person of Indian Origin), and OCB

(Overseas Corporate bodies) with banks authorized by RBI. These can be in the form of savings, current, recurring or fixed deposit accounts. The rate of interest applicable to these accounts is in accordance with the directives issued by RBI from time to time. Statement 2 is incorrect: Repatriation of funds in Foreign Currency (Non-Resident) Account (Banks) or FCNR (B) accounts are permitted. Repartition of Fund is also allowed under the NRE account. While the principal of Non-Resident (Ordinary account) or NRO deposits is non-repatriable, current income and interest-earnings are repatriable. Statement 3 is correct: Repatriation of funds in FCNR (B) and NRE accounts is permitted. Hence, deposits in these accounts are included in India’s external

debt outstanding. While the principal of NRO deposits is non-repatriable, current income and interest-earning are repatriable. Account-holders of NRO accounts are permitted to annually remit an amount up to US$ 1 million out of the balances held in their accounts. Therefore, deposits in NRO accounts too are included in India’s external debt.

Extra Information: Foreign Currency (Non-Resident) Account (Banks) or FCNR (B) accounts can be opened by NRIs, PIOs (Person of Indian Origin), and Overseas Corporate Bodies (OCBs). The accounts can be opened in the form of term deposits only and for a period not less than 1 year and not more than 5 years. The rate of interest applicable to these accounts is in accordance with the directives issued by RBI from time to time. Non-Resident Ordinary Rupee Account Scheme: NRO accounts can be opened by any person resident outside India (Individuals/ entities of Pakistan nationality/ origin and entities of Bangladesh origin require the prior approval of the Reserve Bank of India) with an authorized bank for collecting their funds from local bonafide transactions in Indian Rupees. When a resident becomes an NRI, his existing Rupee accounts are designated as NRO. These accounts can be in the form of current, savings, recurring or fixed deposit accounts. While the principal of NRO deposits is non-repatriable, current income and interest earnings are repatriable. Balances in an NRO account of NRIs/ PIOs are remittable up to USD 1 (one) million per financial year (April-March) along with their other eligible assets.

18. Answer: (d)

Explanation: Statement 1 is incorrect: When the demand for one commodity is related to the price of another commodity is called cross demand. The commodity may be a substitute or complementary. Substitute goods are those goods which can be used

Page 9: BYJU’S IAS PRELIMS TEST SERIES 2021 · 2021. 1. 29. · 2. Answer: (c) Explanation: Recently, a commemorative declaration marking the 75th anniversary of the signing of the Charter

`

8 | SEC-ECO-01

in the case of each other. For example, tea and coffee, Coca-cola and Pepsi. In such cases, demand and price are positively related. This means if the price of one increases then the demand for another also increases and vice versa. For example, a change in the price of tea ordinarily causes a change in demand for coffee. Statement 2 is incorrect: The increase in demand for one particular good causes an increase in the demand for other goods is called derived demand. Complementary goods are those goods which are jointly used to satisfy a want. In other words, complementary goods are those which are incomplete without each other. These are things that go together, often used simultaneously. For example, pen and ink, Tennis rackets and tennis balls, cameras and film, etc.

19. Answer: (a)

Explanation: Recently, a NITI Aayog member has favoured imposing a Border Adjustment Tax. Statement 1 is correct: BAT is a duty that is proposed to be imposed on imported goods in addition to the customs levy that gets charged at the port of entry. To maintain balance between the prices of imported goods and domestically produced goods a country imposes tax at its borders on the same basis and at the same rate as applied on the domestically produced goods. Also, it exempts the taxes on goods exported for sale to foreign consumers. Statement 2 is correct: In general, BAT aims to encourage "equal conditions of

the competition" within the taxing jurisdiction for international and domestic companies providing goods or services. Statement 3 is incorrect: BAT is a fiscal measure that imposes a charge on goods or services in accordance with the destination principle of taxation. Under this principle, the government imposes taxes on products based on the location of their sale to the final consumer rather than

on the location of their production or origin.

20. Answer: (c)

Explanation: The concept of opportunity cost plays an important role in managerial decisions. This concept helps in selecting the best possible alternative from among various alternatives available to solve a particular problem. This concept helps in the best allocation of available resources. Option (a) is incorrect: The change in total cost resulting from a particular decision is known as incremental cost. Option (b) is incorrect: The change in total revenue resulting from a particular decision is known as incremental revenue. Option (c) is correct: The income or benefit foregone as the result of carrying out a particular decision, when resources are limited or when mutually exclusive projects are involved is opportunity cost. Opportunity cost is not what you choose when you make a choice —it is what you did not choose while making a choice. Opportunity cost is the value of the forgone alternative — what you gave up when you got something. For example, if a person has Rs. 100000/- cash in hand, he may think of two options to increase his income. The first option is investing in a bank, where he will get returns of amount Rs.10000. The second option is investing in a business, where he will get returns of amount Rs. 17000. Generally, we choose option 2 because we will get more returns than option 1. Here, option 1 is the opportunity cost. Option (d) is incorrect: The decision by the firm and its impact on both short-run and long-run revenues and cost is known as the Principle of time perspective. A decision by the firm should take into account both short-run and long-run effects on revenues and costs & maintain the right balance between them.

Page 10: BYJU’S IAS PRELIMS TEST SERIES 2021 · 2021. 1. 29. · 2. Answer: (c) Explanation: Recently, a commemorative declaration marking the 75th anniversary of the signing of the Charter

`

9 | SEC-ECO-01

21. Answer: (d)

Explanation: Statement 1 is correct: The Gulf crisis led to a rise in oil prices which is a major import commodity of India. Statement 2 is correct: The Gulf crisis led to the decline of remittances from the gulf region and this led to the decline in the inflow of forex. Statement 3 is correct: Throughout the eighties, all the important indicators of fiscal imbalances were on the rise. These were the conventional budgetary deficit, the revenue deficit, the monetized deficit and gross fiscal deficit.

22. Answer: (d)

Explanation: On 20th June 2020, the Government of India had launched a rural public works scheme - ‘Garib Kalyan Rojgar Abhiyan’ to provide livelihood opportunities to the returnee migrant workers and rural citizens who have returned to their home states due to the Covid-19 induced lockdown.

Statement 1 is incorrect: The priority of the scheme is to satisfy the urgent need for workers who have returned to their districts by providing them with livelihood opportunities. It is a 125-day

based initiative to operate in project mode across 116 districts in six states. The scheme will carry out public works worth 50,000 crore rupees. Statement 2 is incorrect: The scheme covers the migrant workers who returned to 116 districts of six states—Bihar (32 districts), Uttar Pradesh (31), Madhya Pradesh (24), Rajasthan (22), Odisha (4) and Jharkhand (3). These 116 districts also include 27 aspirational districts as identified by the NITI Aayog. The scheme covers not all the aspirational districts but covers aspirational districts within these six states i.e. only 27 (out of 117) aspirational districts are covered.

While both MGNREGA and Garib Kalyan Rojgar Abhiyaan are being implemented by the Ministry of Rural Development, MGNREGA is for 100 days, across all the states and Garib Kalyan Rojgar Abhiyaan is for 125 days and in 6 states only.

23. Answer: (b)

Explanation: Statement 1 is incorrect: When a person quits his current job and is searching for a new job is known as frictional unemployment. Frictional unemployment occurs as a result of people voluntarily changing jobs within an economy. After a person leaves a company, it naturally takes time to find another job. Statement 2 is correct: Introduction of robotic assembly sections in factories leads to loss of jobs for machine operators is known as structural unemployment. Structural unemployment comes about through a technological change in the structure of the economy in which labour markets operate. Technological changes—such as the replacement of horse-drawn transport by automobiles or the automation of manufacturing—lead to unemployment among workers displaced from jobs that are no longer needed. Retraining these workers can be difficult, costly, and time-consuming, and displaced workers often end up either unemployed for extended periods or leaving the labour force entirely. Statement 3 is correct: An agricultural labourer working in fields for 6 months a year and migrating to a city in search of a job for another 6 months is known as seasonal unemployment. Seasonal unemployment occurs when people are unemployed at certain times of the year because they work in industries where they are not needed all year round. Examples of industries where demand, production and employment are seasonal include tourism and leisure, farming, construction and retailing.

Page 11: BYJU’S IAS PRELIMS TEST SERIES 2021 · 2021. 1. 29. · 2. Answer: (c) Explanation: Recently, a commemorative declaration marking the 75th anniversary of the signing of the Charter

`

10 | SEC-ECO-01

24. Answer: (a)

Explanation:

Figure: Lorenz Curve

Source: Investopedia.com

Statement 1 is correct:

Lorenz curve is a graphical way of measuring inequality in income.

It plots the cumulative percentage of total income received against the cumulative percentage of population starting with the poorest individuals.

Statement 2 is incorrect: On the graph, a straight diagonal line represents perfect

equality of wealth distribution. Statement 3 is incorrect: Lorenz curve shows the actual scenario with respect to the wealth distribution. Farther the country’s Lorenz curve from the Perfect Equality Line, higher is the inequality.

25. Answer: (c)

Explanation: In June 2020, the 21st meeting of the Central Employment Guarantee Council was held. Statement 1 is correct: The Central Employment Guarantee Council (CEGC) was constituted under Section 10 of the Mahatma Gandhi National Rural Employment Guarantee Act (MGNREG Act), 2005. Statement 2 is incorrect: The function of this council is to advise the government about the implementation of this scheme and monitoring, redressal mechanism for

this scheme. MGNREGA provides

manual labour in rural areas hence not related to organised sector employment. Statement 3 is correct: The council is chaired by the Minister of Rural Development. The members of this council include both official and non-official members. The body prepares an annual report regarding implementation of this scheme, which is tabled before the parliament.

26. Answer: (c)

Explanation: Statements 1 and 3 are incorrect: The primary sector includes all those activities the end purpose of which consists in exploiting natural resources: agriculture, fishing, forestry, mining, deposits. Pickle making by SHG is considered as a cottage industry which is essentially agriculture and allied sectors. Statements 2 and 4 are correct: The Secondary sector of the economy includes those economic sectors that create finished and usable products: manufacturing like mobiles, construction and electricity generation.

27. Answer: (b)

Explanation: Statements 1 and 2 are correct: As a household's income increases, the percentage of income spent on food decreases while the proportion spent on other goods such as luxury goods, increases. This is because the amount and quality of food a family can consume in a week or month is fairly limited in price and quantity. Statement 3 is incorrect: As per Engel’s law, with higher income, though there is an increase in spending on luxury goods. There is an increase in saving as well, since there is increased net disposable income in the hands of the household.

Page 12: BYJU’S IAS PRELIMS TEST SERIES 2021 · 2021. 1. 29. · 2. Answer: (c) Explanation: Recently, a commemorative declaration marking the 75th anniversary of the signing of the Charter

`

11 | SEC-ECO-01

28. Answer: (a)

Explanation: On the request of Japan and Taiwan, The World Trade Organisation's (WTO) Dispute Settlement Body (DSB) has recently set up two dispute settlement panels targeting import duties imposed by India on a range of ICT goods, including cell phones.

Statement 1 is correct: The ITA is a plurilateral agreement implemented by the World Trade Organization (WTO) and it was concluded in the 1996 Ministerial Declaration on Trade in Products of Information Technology, which entered into force on 1 July 1997. Since then, the number of participants has grown to 82, representing about 97 percent of world trade in IT products. Statement 2 is correct: Participants are committed to the complete removal of tariffs applied to IT goods protected by the Agreement and aims to lower all taxes and tariffs on information technology products to zero. This agreement requires each participant's trade regime to develop in a manner that improves market access opportunities for information technology products. Customs duties and other duties and charges of some sort shall be bound and removed by each participant. The elimination of customs duties for ITA goods, through equal phase rate reductions, began in July 1997 and was completed by the year 2000. Statement 3 is incorrect: India is a signatory to the Information Technology Agreement (ITA) (now also known as ITA-1), a plurilateral agreement of WTO. India joined the ITA on 25th March 1997.

29. Answer: (c)

Explanation: Statement 1 is correct: In economics, the stock of unsold finished goods, or semi-finished goods, or raw materials which a firm carries from one year to the next is called inventory.

Statement 2 is incorrect: A flow variable is measured over an interval of time. It represents the change in the quantity made in a particular time period. And a stock variable is measured at one specific time, and represents a quantity existing at that point in time, which may have accumulated in the past. As inventories are usually measured or evaluated at any single point in time, it is a stock variable. The ‘Inventory Change’ is a flow variable as change takes place over a period of time.

Statement 3 is correct: Inventories are treated as capital. Addition to the stock of capital of a firm is known as investment. As the change in inventories is a flow equal to the change in the stock of unsold goods. Therefore, the change in the inventory of a firm is treated as an investment. The rise in the value of inventories of a firm over a year is treated as investment expenditure undertaken by the firm.

30. Answer: (c)

Explanation:

Statement 1 is Correct: The Harrod Domar Growth model is a growth model. It helps to explain how growth has occurred and how it may occur again in the future. In India, this model was used in the First Five Year Plan. The model suggests that the economy's rate of growth depends on: First, a high level of savings since higher savings enabled greater investment and second, a low capital-output ratio that ensured efficient investment and a higher growth rate.

Statement 2 is Correct: P C Mahalanobis model gave priority to capital goods, as they were crucial for the further economic growth of India. It emphasized basic heavy industries which were adopted first of all in the Second Plan and continued to hold the stage in Indian planning right up to the Fifth Plan. Mahalanobis identifies the rate of growth of investment in the economy not with rate of growth of savings as is usually considered by the economists but with the rate of growth of

Page 13: BYJU’S IAS PRELIMS TEST SERIES 2021 · 2021. 1. 29. · 2. Answer: (c) Explanation: Recently, a commemorative declaration marking the 75th anniversary of the signing of the Charter

`

12 | SEC-ECO-01

output in the capital goods sector within the economy.

31. Answer: (b)

Explanation: In June 2020, Russia declared a state of emergency after a power plant fuel leaked into Ambarnaya River in a sensitive Arctic region. This leak caused 20,000 tonnes of diesel oil to escape into the river.

The power plant is built entirely on permafrost, whose weakening over the years due to climate change caused the pillars supporting a fuel tank to sink. Oil leaked into the local Ambarnaya River in the Arctic region turning the surface

crimson red. The Ambarnaya River flows to the Pyasino Lake and river Pyasina, which connects it to the Kara Sea, a part of the Arctic Ocean.

Extra Information: Permafrost is a ground that remains completely frozen at 0°C or below for at least two years. It is composed of rock, soil and sediments held together by ice and are believed to have formed during glacial periods dating back several millennia.

Figure: Location of Ambarnaya

River on map. Source: bbc.com

32. Answer: (a)

Explanation: The economic cycle is the fluctuation of the economy between periods of expansion (growth) and contraction (recession). Factors such as gross domestic product (GDP), interest rates, total employment, and consumer spending, can help to determine the current stage of the economic cycle.

Figure: Graphical representation of Business Cycle or Economic Cycle

Source: Investopedia

The four stages are expansion, peak, contraction, and trough.

Statement 1 is correct: During the boom phase, the growth of an economy is at a maximum rate thus keeping the unemployment low. It is usually characterized by higher inflation. Statement 2 is correct: During the recovery phase, the economy experiences relatively rapid growth, interest rates tend to be low, production increases, and inflationary pressures build. Statement 3 is correct: During the contraction phase, wherein the growth of the economy slows, unemployment rates rise, and inflation comes down. Statement 4 is incorrect: The trough is the bottom of the recession period, unemployment is at its highest, inflation is low.

Page 14: BYJU’S IAS PRELIMS TEST SERIES 2021 · 2021. 1. 29. · 2. Answer: (c) Explanation: Recently, a commemorative declaration marking the 75th anniversary of the signing of the Charter

`

13 | SEC-ECO-01

33. Answer: (b)

Explanation: MSS bonds are special bonds floated on behalf of the government by the Reserve

Bank of India for the sole purpose of absorbing the excess liquidity in the system by selling government bonds, when normal government bonds become inadequate. The Reserve Bank first launched MSS bonds in February 2004 when the country was flushed by dollar inflows, which had to be transformed into rupee. This created a huge surplus of liquidity in the economy, and the RBI decided to absorb it by issuing MSS bonds, as the central bank was running out of regular government bonds.

Statement 1 is correct: These are often short-tenure bonds with a maturity of

fewer than six months. But depending on the criteria, the tenure varies. The primary objective of this scheme is to help the RBI's sterilisation operations. Statement 2 is correct: MSS bonds are raised through an auction and are tradable in the secondary market. Statement 3 is incorrect: MSS bonds earn a return and qualify for statutory liquidity ratio (SLR) that banks need to maintain in the form of short-tenured treasury bills and government bonds.

Extra information: Regular government bonds are a part of the borrowing scheme of the government and the payment of interest on them affects the fiscal situation. In the case of MSS bonds, an equal cash balance kept by the government with the Reserve Bank matches the MSS bills and securities. They have, therefore, just a marginal effect on the revenue and fiscal positions of the country. The cost of the payment of such interest is shown separately in the budget.

34. Answer: (c)

Explanation: Statement 1 is incorrect: CPI-IW is compiled and maintained by the Labour Bureau, an attached office of the Ministry of Labour & Employment. Statement 2 is correct: The labour ministry has revised the base year of the Consumer Price Index-Industrial Workers (CPI-IW) from 2001 to 2016. The revision in the base year will reflect the changing consumption pattern of the working-class population over the years. Following the change in the base year, the index will give 39% weight to food and beverage consumption of workers now as against 46.2% earlier. In contrast, spending on housing will get almost 17% weight as against 15.2% earlier. Statement 3 is correct: The CPI-IW is used as a benchmark for calculating dearness allowance for government employees, dearness relief for pensioners and wages for industrial workers in some sectors.

35. Answer: (d)

Explanation: In June 2020, India joined GPAI as a founding member. Statement 1 is correct: GPAI is a multi-stakeholder international partnership to promote responsible and human-centric development and use of AI, grounded in human rights, inclusion, diversity, innovation and economic growth. This is the first initiative of its type for evolving better understanding of the challenges and opportunities around AI using the experience and diversity of participating countries. Statement 2 is correct: GPAI is supported by a Secretariat hosted by Organization for Economic Cooperation and Development (OECD) in Paris, and by two Centers of Expertise- one each in Montreal and Paris.

Page 15: BYJU’S IAS PRELIMS TEST SERIES 2021 · 2021. 1. 29. · 2. Answer: (c) Explanation: Recently, a commemorative declaration marking the 75th anniversary of the signing of the Charter

`

14 | SEC-ECO-01

Statement 3 is correct: India is a founding member of this organization. By joining GPAI as a founding member, India will actively participate in the global development of Artificial Intelligence, leveraging its experience in the use of digital technologies for inclusive growth.

36. Answer: (b)

Explanation: Demand-pull inflation occurs when aggregate demand in an economy outpaces aggregate supply. To tackle the demand-pull inflation, liquidity should be reduced. Statement 1 is correct: Higher taxes would reduce the money with the public and as a result the demand for goods and services would be reduced. Hence, the introduction of Swachh Bharat cess could be a method to combat demand-pull inflation. Statement 2 is incorrect: Printing extra money will increase the money in the economy and that means more money in the hands of people which will increase the demand-pull inflation. Statement 3 is incorrect: Introducing a new line of subsidies, will form part of government expenditure to provide more money in hands of people. Also, if the government undertakes deficit financing to achieve this, it increases the flow of money in the market and thereby increases inflation.

37. Answer: (b)

Explanation: Statement 1 is incorrect: Changed the base year from 2004-05 to 2011-12. Statement 2 is correct: GDP at market prices is included in the calculation of headline GDP instead of GDP at factor cost. Statement 3 is incorrect: Use of data from the MCA 21 database instead of data from earlier Annual Survey of Industries and

Index of Industrial Production data to estimate the GDP of enterprises.

38. Answer: (b)

Explanation: According to the recent NCLAT’s judgment, if a corporate debtor is being liquidated, a creditor can not claim superiority over other secured creditors in the same band. It says that everyone must receive their fair share by following the waterfall mechanism of liquidation. Option (a) is incorrect: Open Acreage Licensing Policy (OALP) is a revenue-sharing model under which uniform licenses are provided for exploration and production of all forms of hydrocarbons. It is a critical part of Hydrocarbon Exploration and Licensing Policy. Option (b) is correct: The waterfall mechanism mentioned under Section 53 of the Insolvency and Bankruptcy Code of India outlines the order of priority for repayment to creditors in the event of a liquidation. Under this, secured creditors have to be paid fully before any payments can be made to unsecured financial creditors who in turn have priority over operational creditors. Option (c) is incorrect: Swiss challenge method is a new process of giving contracts. Any person with credentials can submit a development proposal to the government. That proposal will be made online and a second person can give suggestions to improve and beat that proposal and an expert committee will accept the best proposal and the original proposer will get a chance to accept it if it is an improvement over the original proposal. Option (d) is incorrect: Haulage Policy is a mechanism through which private operators of Tejas trains are to be charged for the usage of railway infrastructure.

Page 16: BYJU’S IAS PRELIMS TEST SERIES 2021 · 2021. 1. 29. · 2. Answer: (c) Explanation: Recently, a commemorative declaration marking the 75th anniversary of the signing of the Charter

`

15 | SEC-ECO-01

39. Answer: (b)

Explanation: "SATYABHAMA (Science and Technology Yojana for Atmanirbhar Bharat in Mining Advancement)" is a portal launched by the Ministry of Mines, with an aim to promote research and development in the mining and minerals sector. This portal allows online submission of project proposals, monitoring and utilisation of funds.

40. Answer: (c)

Explanation: Statement 1 is correct: The national income estimation is the responsibility of the National Statistics Organization (NSO) which comes under the Ministry of Statistics and Program Implementation. Statement 2 is incorrect: There are three ways to calculate a country’s gross domestic product. And, in theory, no matter what method you use — the result should be the same value. Due to non-availability of the data, no single method can solely be used in India. We use a mixture of all three methods, which are Production Method, Expenditure Method and Income Method. To assess India’s productivity, the GDP is calculated using the factor cost method across eight industries and the expenditure method is used to analyse how different areas of the economy are performing. Statement 3 is correct: India adopted 'The System of National Accounts 2008' in the year 2015 and thereby revised the GDP base year from 2004-05 to 2011-12. The United Nations Statistical Commission adopted 'The System of National Accounts 2008' after intense discussions and consultation with member countries in 2009.

41. Answer: (b)

Explanation: Inflation Premium: The bonus brought by inflation to the borrowers is known as Inflation Premium. The interest banks charge on their lending is known as the nominal interest rate, which might not be the real cost of borrowing paid by the borrower to the banks. Rising inflation premium shows depleting profits of the lending institutions. At times, to neutralise the effects of inflation premium, the lender takes the recourse to increase the nominal rate of interest.

Statement 1 is incorrect: To neutralise the effects of Inflation Premium, the lender should increase the nominal rate of interest. Statement 2 is correct: To neutralise the effects of Inflation Premium, the RBI should follow a tight credit policy.

42. Answer: (c)

Explanation: Statement 1 is incorrect: EHe is a low-mass supergiant star that has a negligible amount of hydrogen and has an abundance of surface helium. This property of EHe is in contrast to the majority of Stars (including Sun) which contain around 70% hydrogen (by mass) throughout their lives. Statement 2 is correct: Despite having lesser mass, EHe stars are much larger and hotter than the Sun. Statement 3 is correct: EHe was in the news recently as the presence of singly ionized fluorine was detected in the atmosphere. The main formation of EHe involves a merger of a carbon-oxygen and a Helium (He) white dwarf.

Page 17: BYJU’S IAS PRELIMS TEST SERIES 2021 · 2021. 1. 29. · 2. Answer: (c) Explanation: Recently, a commemorative declaration marking the 75th anniversary of the signing of the Charter

`

16 | SEC-ECO-01

43. Answer: (a)

Explanation: India framed a Model BIT in 2016, moving away from an overly investor-friendly approach to a somewhat protectionist approach concerning foreign investments. Since its adoption, India has unilaterally terminated 66-odd BITs between 2016 to 2019, sending negative signals to the global investor community. The essential features of the model BIT include an "enterprise" based definition of investment, non-discriminatory treatment through due process, national treatment, protections against expropriation, a refined Investor-State Dispute Settlement (ISDS) provision requiring investors to exhaust local remedies before commencing international arbitration, and limiting the power of the tribunal to awarding monetary compensation alone. The model excludes matters such as government procurement, taxation, subsidies, compulsory licenses and national security to preserve the regulatory authority for the Government. Statement 1 is correct: BITs typically serve to protect investments made by investors on a reciprocal basis, specifying conditions on regulatory oversight of the host state and limiting interference with the rights of foreign investors. Statement 2 is correct: It contains a clause mandating exhaustion of domestic remedy prior to initiating international arbitration proceedings under ISDS mechanism. According to the ‘Ease of Doing Business 2020’ report, India currently ranks 163 out of 190 countries in ease of enforcing contracts, and it takes 1,445 days and 31 per cent of the claim value for dispute resolution. Statement 3 is incorrect: Model BIT has done away with the ‘Fair and Equitable

Treatment’ clause and has included a detailed ‘Treatment of Investments’ clause with a broadly-worded undertaking that neither party shall subject investments to measures that are manifestly abusive, against norms of customary international

law and to un-remedied and egregious violations of due process.

44. Answer: (c)

Explanation: Statement 1 is correct: The Reserve Bank of India (RBI) recently announced the creation of a Rs 500-crore Payments Infrastructure Development Fund (PIDF). Statement 2 is correct: It aims to encourage Points of Sale (PoS) infrastructure (both physical and digital modes), in tier-3 to tier-6 centres and northeastern states. Statement 3 is incorrect: The Reserve Bank will make an initial contribution of ₹250 crores to the PIDF covering half the fund and remaining contribution will be from the card-issuing banks and card networks operating in the country. The PIDF will also receive recurring contributions to cover operational expenses from the card-issuing banks and card networks. The Reserve Bank will also contribute to yearly shortfalls, if necessary.

Hence, NPCI has no role in either finance or administration of PIDF.

45. Answer: (c)

Explanation: Statement 3: Visvesvaraya Plan (1934) - Sir M Visvesvaraya published his book “Planned Economy in India”, in which he presented a constructive draft of the development of India in 10 years. He planned for shifting labour from agriculture to industries and to double the National income in 10 years. Statement 4: Gandhian Plan (1944) - This plan was put forward by Shriman Narayan Aggarwal in 1944 who was a principal of Wardha Commercial College. It emphasized on cottage industries and self-contained villages. Statement 2: Congress Plan (1948): The National Planning Committee was set up in 1938 by Subhash Chandra Bose and chaired by Jawaharlal Nehru. But, the reports of the committee could not be

Page 18: BYJU’S IAS PRELIMS TEST SERIES 2021 · 2021. 1. 29. · 2. Answer: (c) Explanation: Recently, a commemorative declaration marking the 75th anniversary of the signing of the Charter

`

17 | SEC-ECO-01

prepared and only for the first time in 1948 - 49 some papers came out. Statement 1: Sarvodaya Plan (1950): It was drafted by Jayprakash Narayan inspired by Gandhian plan as well as Sarvodaya idea of Vinoba Bhave. It emphasized both on small and medium cotton industries and agriculture as well. This plan suggested freedom from foreign technology and stressed upon land reforms and decentralized participatory planning.

Extra Information: Bombay Plan (1944): Eight Industrialists of Bombay including Mr JRD Tata, GD Birla, Purshottamdas Thakurdas and others prepared “A Brief Memorandum Outlining a Plan of Economic Development for India”. This was popularly known as the Bombay Plan. It envisaged doubling the per capita income in 15 years and tripling the national income during this period.

46. Answer: (d)

Explanation: In accordance with Section 47 of the RBI Act, profits or surpluses of the RBI are to be transferred to the government after various contingency arrangements have been made, including financial stability considerations, of the RBI's public policy mandates. Statement 1 is correct: Contingency Fund (CF) is designed to meet unexpected and unforeseen contingencies, including the deterioration of the value of assets, the risk of monetary/exchange rate policy operations, structural risks and any risk resulting from the special obligations placed on the Reserve Bank. Statement 2 is correct: The Reserve Bank maintains that it takes control of currency risk, interest rate risk and gold price movement. A buffer against exchange rate/gold price fluctuations is given by the Currency and Gold Revaluation Account (CGRA). If there is an appreciation of the rupee against major

currencies or a fall in the price of gold, it may come under pressure. Statement 3 is correct: In the IRA-FS account, the unrealized gains or losses on the revaluation of foreign dated securities are reported. Statement 4 is correct: The unrealized revaluation gains or losses are recorded in the Investment Revaluation Account-Rupee Securities (IRA-RS).

47. Answer: (d)

Explanation: According to a Deloitte report, the female labour force participation in India has fallen to 26% in 2018 from 36.7% in 2005. Statement 1 is correct: Insufficient job opportunities due to factors such as social status and occupational segregation due to which women are often discouraged to pursue or enter roles and positions such as in combat positions in the army, or factory jobs that are thought to be male-dominated. Statement 2 is correct: Significantly a higher percentage of women tend to

pursue higher degrees and accolades, thus increasing their duration of academics. Statement 3 is correct: Increased household income is also a reason that women are not looking for jobs in both rural and urban areas Statement 4 is correct: The decline in the number of women in the workforce is accounted for mainly by rural women, which can be attributed to the stagnant agricultural sector. The manufacturing sector hasn’t become robust enough to absorb the job losses caused by the decline in the agricultural sector.

Page 19: BYJU’S IAS PRELIMS TEST SERIES 2021 · 2021. 1. 29. · 2. Answer: (c) Explanation: Recently, a commemorative declaration marking the 75th anniversary of the signing of the Charter

`

18 | SEC-ECO-01

48. Answer: (d)

Explanation: Statement 1 is correct: The concept of the SDGs was born at the United Nations Conference on Sustainable Development, Rio+20, in 2012. The Sustainable Development Summit was held from 25-27 September 2012 at UN Headquarters in New York. Countries officially adopted the historic new agenda, entitled “Transforming Our World: The 2030 Agenda for Sustainable Development,” which was agreed upon by the 193 Member States of the United Nations and includes 17 Sustainable Development Goals (SDGs). These SDG targets have succeeded the Millennium Development Goals (MDG). Statement 2 is correct: NITI Aayog has been assigned the role of overseeing the implementation of SDGs in the country. For this, the SDG India Index was developed by NITI Aayog. The implementation of Sustainable Development Goals (SDGs) has started from 1 January 2016. Statement 3 is correct: The objective was to produce a set of universally applicable goals that balances the three dimensions of sustainable development: environmental, social, and economic. There are a total of 17 SDGs and 169 associated targets.

49. Answer: (c)

Explanation: The Mahatma Gandhi National Rural Employment Guarantee Act, earlier known as the National Rural Employment Guarantee Act was passed on 7th September 2005 to augment employment generation and social security in India. It provides 100 days of guaranteed wage employment to rural unskilled labour. Statement 1 is incorrect: Unlike earlier employment guarantee schemes, the Mahatma Gandhi National Rural Employment Guarantee Act, 2005 provides a legal right to employment for

adult members of rural households. Anyone who is a member of a registered household in the village, who is above the age of 18 years and is willing to take unskilled work has a Right to get a Job Card and work under MGNREGA. Statement 2 is correct: Under MGNREGA 2005, all work sites should have facilities such as crèches, drinking water and first aid. Statement 3 is correct: Social Audit is the examination and assessment of a programme/scheme conducted with the active involvement of people and comparing official records with actual ground realities. Social audits are conducted by gram sabhas to enable the community to monitor the implementation of the scheme. Section 17 of the MGNREGA has mandated social audit of all works executed under the MGNREGA.

50. Answer: (b)

Explanation: There are four main aggregate expenditures that go into calculating GDP: consumption by households, investment by businesses, government spending on goods and services and net exports, which are equal to exports minus imports of goods and services. Statement 1 is correct: An individual spending food items is part of the consumption expenditure on the goods and services produced by a firm. Statement 2 is incorrect: A company buying raw material for manufacturing is expenditure on intermediate goods, which is not included in the calculation of GDP because intermediate goods are consumed in the process of production. Statement 3 is correct: Government expenditure on defence is the expenditure that the government makes in the economy. Hence, it is factored in while calculating GDP.

Page 20: BYJU’S IAS PRELIMS TEST SERIES 2021 · 2021. 1. 29. · 2. Answer: (c) Explanation: Recently, a commemorative declaration marking the 75th anniversary of the signing of the Charter

`

19 | SEC-ECO-01

51. Answer: (a)

Explanation: Statement 1 is correct: The Industrial Policy Resolution of 1956 formed the basis of the Second Five Year Plan. The second five-year plan tried to build the basis for a socialist pattern of society. Statement 2 is correct: The purpose of this policy was to promote regional equality. Hence, it became the basis for promoting industries in the backward regions. Statement 3 is incorrect: This resolution classified industries into three categories.

The first category comprised industries which would be exclusively owned by the state;

The second category consisted of industries in which the private sector could supplement the efforts of the state sector, with the state taking the sole responsibility for starting new units;

The third category consisted of the remaining industries which were to be in the private sector.

52. Answer: (c)

Explanation: Statement 1 is incorrect: The LTRO is a tool under which the central bank provides one-year to three-years money to banks at the prevailing repo rate, accepting government securities with matching or higher tenure as the collateral. Statement 2 is correct: LTRO helped RBI ensure that banks reduce their marginal

cost of funds-based lending rate, without reducing policy rates. i.e, as banks get long-term funds at lower rates, their cost of funds falls. In turn, they reduce interest rates for borrowers. Therefore, better transmission of monetary policy is achieved. Statement 3 is correct: LTRO enhances liquidity in the system, which will bring down the short term interest rates thus bringing down the costs in the corporate

bond market. These new measures coupled with RBI’s earlier introduced ‘Operation Twist’ are an attempt by the central bank to manage bond yields and push transmission of earlier rate cuts.

53. Answer: (a)

Explanation: In Call Money Market (CMM) the market loans can be availed by banks to meet liquidity for overnight (one day). The call money rate is an important variable for the RBI to assess the liquidity situation in the economy. The CMM is known as the most sensitive segment of the financial system. Statement 1 is correct: Call money is a short-term, interest-paying loan given by one financial institution to another, for a very short period without any collateral. Statement 2 is incorrect: The call money market interest rates are not determined by RBI, but the market itself. Statement 3 is incorrect: In the call money market, banks and primary dealers (PDs) are allowed to both borrow and lend. But non-bank participants such as financial institutions, mutual funds and select corporates are permitted to only lend. Hence, they are permitted to participate.

54. Answer: (b)

Explanation: Ease of Living Index is aimed at providing a holistic view of Indian cities: beginning from the services provided by local bodies, the effectiveness of the administration, the outcomes generated through these services in terms of the liveability within cities and, finally, the citizen perception of these outcomes. The key objectives of the Ease of Living Index are: 1. To generate information to guide

evidence-based policy making. 2. To catalyse action to achieve broader

developmental outcomes including the SDGs.

Page 21: BYJU’S IAS PRELIMS TEST SERIES 2021 · 2021. 1. 29. · 2. Answer: (c) Explanation: Recently, a commemorative declaration marking the 75th anniversary of the signing of the Charter

`

20 | SEC-ECO-01

3. To assess and compare the outcomes achieved from various urban policies and schemes.

4. To obtain the perception of citizens about their views with respect to the services provided by the city administration.

Statement 1 is incorrect: Ease of Living Index (EoLI) is launched by the Ministry of Housing & Urban Affairs. Statement 2 is correct: Ease of Living Index (EoLI) facilitates the assessment of ease of living of citizens across three pillars: Quality of Life, Economic Ability and Sustainability which are further divided into 14 categories across 50 indicators. Statement 3 is correct: For the first time in 2020, as a part of the Ease of Living Index Assessment, a Citizen Perception Survey was conducted on behalf of the Ministry (which carries 30% of the marks of the Ease of Living Index). This is a very important component of the assessment exercise as it will help in directly capturing perception of citizens with respect to quality of life in their cities.

55. Answer: (a)

Explanation: Payments Banks are a new set of banks authorised by Reserve Bank of India for furthering the financial inclusion in India.

Statement 1 is correct: The following are eligible to set up payment banks:

Non-Banking Finance Companies (NBFCs)

Corporate Banking Correspondents (BCs)

Mobile telephone companies & Supermarket chains.

Public sector entities The banks can take an equity stake in a Payments Bank to the extent permitted under the Banking Regulation Act, 1949.

Statement 2 is correct: Payment banks can take up the following functions:

They can provide small savings/ current accounts for deposits below Rs. 1 lakh.

They can distribute mutual funds, insurance products on a non-risk sharing basis, and can undertake payments/remittance services.

Payments Bank cannot undertake lending business nor can provide fixed deposit services

Payment Banks cannot set up subsidiaries to undertake non-banking activities in the field of financial services.

They can issue debit cards but not credit cards.

They are not allowed to accept NRI deposits.

Statement 3 is incorrect: They are bound by the reserve requirement rules of RBI (CRR, SLR etc.). They are required to invest a minimum 75 per cent of their "demand deposit balances'' in Statutory Liquidity Ratio (SLR) eligible Government securities/treasury bills. They also need to keep up to 25 percent of current and time/fixed deposits with other scheduled commercial banks for operational and liquidity management purposes.

56. Answer: (c)

Explanation: Statement 1 is correct: Employment elasticity is a measure of the percentage change in employment associated with a 1 percentage point change in economic growth. Statement 2 is incorrect: Additional labour force can be employed only when employment elasticity of growth is high. If the employment elasticity of growth is less, then some of the labour force remains employed. Statement 3 is incorrect: The high GDP growth rate can not always reflect the higher employment elasticity. After 2000 India experienced a jobless growth phenomenon, in which growth rate was as high as 8% but the employment elasticity was at a low level. For example, between

Page 22: BYJU’S IAS PRELIMS TEST SERIES 2021 · 2021. 1. 29. · 2. Answer: (c) Explanation: Recently, a commemorative declaration marking the 75th anniversary of the signing of the Charter

`

21 | SEC-ECO-01

2004–05 to 2009–10, employment elasticity of India was as low as 0.01.

57. Answer: (d)

Explanation: Statement 1 is correct: A higher GDP growth rate does not mean all the people in the country are growing at the same rate. Many people might be too poor. But for others, earning could be much higher leading to a skewed GDP growth rate. Statement 2 is correct: Informal productive activities carried on by barter system or domestic services provided by women and men are not considered. This leads to undercounting. Statement 3 is correct: Harmful externalities like pollution and environmental degradation are not accounted for in the calculation of GDP. Extra Information Green accounting is a type of accounting that attempts to factor environmental costs into the financial results of operations. Gross Domestic Product ignores the environmental cost and therefore policymakers need to incorporate a revised model that incorporates green accounting.

58. Answer: (a)

Explanation: Statement 1 is correct: An escrow account is a third party account where funds are kept before they are transferred to the ultimate party. It provides security against scams and frauds especially with high asset value and dispute-prone sectors like Real Estate. Statement 2 is incorrect: Escrow accounts can hold money, securities, funds, and other assets. Statement 3 is incorrect: The party receiving the money can not make withdrawals from an escrow account until the set obligations are fulfilled. Hence, this

proves to be a powerful financial instrument that removes trust-based issues in sectors like Real Estate.

59. Answer: (a)

Explanation: Statement 1 is correct: Micro-insurance policies are a special type of insurance policies that are intended to encourage insurance coverage for economically disadvantaged sections of the society. Statement 2 is correct: These policies were introduced by the Insurance Regulatory Development Authority of India (IRDA). Statement 3 is incorrect: Microinsurance may be either a general insurance policy (which may insure health, household, houses, tools, personal accident injury, livestock, etc.) or a life insurance policy with an amount insured of Rs 50,000 or less. Insurers can offer composite covers or package products that include life and general insurance covers together. Statement 4 is incorrect: Microinsurance business can be done through the following intermediaries in India: Non-Government Organisations, Self-Help Groups, Micro-Finance Institutions.

60. Answer: (a)

Explanation: Statement 1 is incorrect: The part of the final output that comprises capital goods constitutes the gross investment of an economy. New capital formation is the new addition to the capital stock in an economy. Statement 2 is correct: New addition to the capital stock in an economy is measured by net investment or new capital formation, which is expressed as

Net Investment = Gross investment – Depreciation.

Statement 3 is correct:

National Disposable Income = Net National Product at market prices +

Page 23: BYJU’S IAS PRELIMS TEST SERIES 2021 · 2021. 1. 29. · 2. Answer: (c) Explanation: Recently, a commemorative declaration marking the 75th anniversary of the signing of the Charter

`

22 | SEC-ECO-01

other current transfers from the rest of the world.

The idea behind National Disposable Income is that it gives an idea of what is the maximum amount of goods and services the domestic economy has at its disposal.

Current transfers include workers' remittances, donations, tax payments, foreign aid, and grants.

The current transfers include all transfers that do not have the following characteristics of capital transfers: Transfers of ownership of fixed assets. Transfers of funds linked to acquisition or disposal of fixed assets.

61. Answer: (d)

Explanation: Statement 1 is incorrect: The Periodic Labour Force Survey (PLFS) is released by the National Statistical Office (NSO) under the Ministry of Statistics and Programme Implementation. It was first published in 2017. Statement 2 is correct: The labour force participation rate (LFPR) during 2018-19 rose to 37.5 percent as compared to 36.9 percent a year ago, said the Periodic Labour Force Survey (PLFS), released by the Ministry of Statistics and Programme Implementation. Statement 3 is correct: The Periodic Labour Force Survey (PLFS) gives estimates of Key employment and unemployment Indicators like the Labour Force Participation Rates (LFPR), Worker Population Ratio (WPR), Unemployment Rate (UR), etc. These indicators are defined as follows: 1. Labour Force Participation Rate

(LFPR): LFPR is defined as the percentage of persons in the labour force (i.e. working or seeking or available for work) in the population.

2. Worker Population Ratio (WPR): WPR is defined as the percentage of employed persons in the population.

3. Unemployment Rate (UR): UR is defined as the percentage of persons

unemployed among the persons in the labour force.

4. Activity Status- Usual Status: The activity status of a person is determined based on the activities pursued by the person during the specified reference period. When the activity status is determined based on the reference period of the last 365 days preceding the date of the survey, it is known as the usual activity status of the person.

5. Activity Status- Current Weekly Status (CWS): The activity status determined based on a reference period of the last 7 days preceding the date of the survey is known as the current weekly status (CWS) of the person.

62. Answer: (b)

Explanation: Priority Sector Lending is an important role given by the (RBI) to the banks for providing a specified portion of the bank lending to few specific sectors like agriculture and allied activities, micro and small enterprises, poor people for housing, students for education and other low-income groups and weaker sections. This is essentially meant for the all-round development of the economy as opposed to focusing only on the financial sector. Statement 1 is incorrect: Foreign banks with less than 20 branches in India have been brought under the ambit of targets under Priority Sector lending (PSL). Statement 2 is incorrect: It is one of the qualitative tools of monetary policy of the Reserve Bank of India. Statement 3 is correct: Bank credit to MFIs is considered as a part of Priority Sector Lending (PSL). Bank credit to MFIs (NBFC-MFIs, societies, trusts, etc) extended for on-lending to individuals and also to members of SHGs/JLGs is eligible for categorisation as priority sector advance under respective categories viz., Agriculture, Micro, Small and Medium Enterprises, Social Infrastructure.

Page 24: BYJU’S IAS PRELIMS TEST SERIES 2021 · 2021. 1. 29. · 2. Answer: (c) Explanation: Recently, a commemorative declaration marking the 75th anniversary of the signing of the Charter

`

23 | SEC-ECO-01

63. Answer: (d)

Explanation: The debt-service coverage ratio applies to corporate, government, and personal finance.

In the context of corporate finance, the debt-service coverage ratio (DSCR) is a measurement of a firm's available cash flow to pay current debt obligations. The DSCR shows investors whether a company has enough income to pay its debts.

In the context of government finance, the DSCR is the amount of export earnings needed by a country to meet annual interest and principal payments on its external debt.

In the context of personal finance, it is a ratio used by bank loan officers to determine income property loans.

Statement 1 is correct: The ratio states net operating income as a multiple of debt obligations due within one year, including interest, principal, sinking funds, and lease payments. DSCR of less than 1 means a borrower will be unable to cover or pay current debt obligations without drawing on outside sources. For example, a DSCR of 0.95 means that there is only sufficient net operating income to cover 95% of annual debt payments. Statement 2 is correct: The debt service coverage ratio is a benchmark used to measure the income of an individual to cover his/her debt payments. A higher debt service coverage ratio makes it easier to obtain a loan. Hence, it is used by bank loan officers to determine income property loans. Statement 3 is correct: In the context of government finance, the DSCR is the amount of export earnings needed by a country to meet annual interest and principal payments on its external debt.

64. Answer: (b)

Explanation: A non-banking institution or Non-Banking Finance Company (NBFCs), is a company that has principal business of receiving deposits under any scheme or arrangement in one lump sum or installments by way of contributions or in any other manner. Statement 1 is incorrect: NBFCs are registered under the Companies Act, 1956 (amended in 2013), not under the Banking Regulation Act, 1949. Statements 2 and 3 are correct: NBFCs lend and make investments and thus their practices are close to those of banks; however, there are a few distinctions as set out below:

NBFC cannot accept demand deposits.

NBFCs do not form part of the payment and settlement system and cannot issue cheques on their own.

Deposit Insurance and Credit Guarantee Corporation's deposit insurance facility is not open to NBFCs depositors, as is the case for banks.

65. Answer: (a)

Explanation: The Index of Industrial Production is published monthly by the Central Statistics Organization. It follows a six weeks lag due to data compilation and processing. It calculates the data of eight core sectors.

Weightage of industries: 1. Petroleum Refinery production - 28.04

percent 2. Electricity generation - 19.85 percent 3. Coal production - 10.33 percent 4. Fertilizers production - 2.63 percent

Weightage of other industries: Steel production - 17.92 percent Crude Oil production - 8.98 percent Natural Gas production - 6.88 percent Cement production - 5.37 percent

Page 25: BYJU’S IAS PRELIMS TEST SERIES 2021 · 2021. 1. 29. · 2. Answer: (c) Explanation: Recently, a commemorative declaration marking the 75th anniversary of the signing of the Charter

`

24 | SEC-ECO-01

Extra information:

Since April 2014, Electricity generation data from Renewable sources are also included.

Since March 2019, a new steel product called Hot Rolled Pickled and Oiled (HRPO) under the item ‘Cold Rolled (CR) coils’ within the production of finished steel has also been included.

66. Answer: (d)

Explanation: Cash Reserve Ratio (CRR) is a specified minimum fraction of the total deposits of customers, which commercial banks have to hold as reserves either in cash or as deposits with the central bank. CRR is set according to the guidelines of the central bank of a country. Statement 1 is incorrect: CRR is the ratio of NDTL (net demand and time liabilities) that banks have to maintain with the RBI. Statement 2 is incorrect: It is maintained in the form of cash only and not in the form of gold and other Securities approved by the RBI. Statement 3 is incorrect: Banks don’t receive any interest in maintaining CRR. The Narasimham Committee, in 1991 had suggested that RBI should pay interest on maintaining CRR.

67. Answer: (a)

Explanation: Wholesale banking is the financial practice of lending and borrowing between two large institutions. These types of services are provided by investment banks that often also offer retail banking. This means that an individual looking for wholesale banking wouldn't have to go to a special institution and could instead engage the same bank in which he conducts his retail banking. Wholesale Banking Services are for entities that require more services than normal clients of retail banks. Because of the large scale, the prices offered for

these services are typically lower than what is offered to an individual.

Statement 1 is correct: Wholesale banking refers to banking services sold to large clients, such as other banks, other financial institutions, government agencies, large corporations, and real estate developers. Statement 2 is correct: Wholesale banking services include currency conversion, working capital financing, large trade transactions, mergers and acquisitions, consultancy, and underwriting, among other services. Statement 3 is incorrect: Though it is the opposite of retail banking, which focuses on individual clients and small businesses, these services are also provided to the High Net worth Individuals.

68. Answer: (a)

Explanation: Statement 1 is correct: Any official means of payment recognised by the law of the land shall be referred to by legal tender. It is used to carry out monetary transactions, to comply with a financial obligation or to repay public or private debt. The value of the currency notes and coins is derived from the guarantee provided by the issuing authority. The legal tender does not have intrinsic value like gold or silver coin. Statement 2 is correct: Every banknote issued by Reserve Bank of India (₹2, ₹5, ₹10, ₹20, ₹50, ₹100, ₹200, ₹500 and ₹2000), unless withdrawn from circulation, shall be legal tender at any place in India and shall be guaranteed by the Central Government, subject to provisions of RBI Act, 1934. Statement 3 is incorrect: As per The Coinage Act, 2011, coins of any denomination not lower than one rupee shall be legal tender for any sum not exceeding one thousand rupees. But ₹1 notes issued by the Government of India are legal tender without any limitation.

Page 26: BYJU’S IAS PRELIMS TEST SERIES 2021 · 2021. 1. 29. · 2. Answer: (c) Explanation: Recently, a commemorative declaration marking the 75th anniversary of the signing of the Charter

`

25 | SEC-ECO-01

69. Answer: (b)

Explanation: Option (a) is correct: RBI is the independent authority for conducting monetary policy in the best interests of the economy. It increases or decreases the supply of high powered money in the economy and creates incentives or disincentives for the commercial banks to give loans or credits to investors. Option (b) is incorrect: RBI purchases government securities from the general public in a bid to increase the stock of high powered money in the economy. On the other hand, it sells government securities to reduce the same. Option (c) is correct: RBI can affect the reserve deposit ratio of commercial banks by adjusting Statutory Liquidity Ratio (SLR). A high value of CRR or SLR helps increase the value of reserve deposit ratio, thus diminishing the value of the money multiplier and money supply in the economy. Option (d) is correct: When there are foreign investments in the country, the person who sells these bonds to foreign investors will exchange her foreign currency holding into rupee at a commercial bank. The bank, in turn, will submit this foreign currency to RBI and its deposits with RBI will be credited with equivalent sums of money. Hence, there is an increased supply of money in the economy which leads to inflation. RBI will undertake an open market sale of government securities of an amount equal to the amount of foreign exchange inflow in the economy, thereby keeping the stock of high powered money and total money supply unchanged. This operation of RBI is known as sterilisation. Thus, it sterilises the economy against adverse external shocks.

70. Answer: (b)

Explanation: Bombay Plan proposed trebling of national income and doubling of per capita income within 15 years by giving top priority to industrialisation, especially the production of power and capital goods. Statement 1 is incorrect: A key principle of the Bombay Plan was that the economy could not grow without government intervention and regulation. Under the assumption that the fledgling Indian industries would not be able to compete in a free-market economy, the Plan proposed that the future government protects indigenous industries against foreign competition in local markets. Statement 2 is correct: Large-scale measures for social welfare were favoured by these plans, which suggested focussing on issues like, right to work and full employment, the guarantee of a minimum wage, greater state expenditure on housing, water and sanitation, free education.

71. Answer: (d)

Explanation: Statement 1 is incorrect: Rural Infrastructure Development Fund (RIDF) is maintained by the National Bank for Agriculture and Rural Development (NABARD). Small Enterprises Development Fund (SEDF) was set up and is maintained by Small Industries Development Bank of India (SIDBI). Statement 2 is incorrect: The shortfalls in the priority sector lending of public sector banks are contributed to the Rural

Infrastructure Development Fund (RIDF) established under NABARD. The foreign banks having shortfall in lending to stipulated priority sector targets/sub-targets are required to contribute to SEDF. SEDF is used for the promotion, financing and development of Micro, Small and Medium Enterprises (MSME) sector in India.

Page 27: BYJU’S IAS PRELIMS TEST SERIES 2021 · 2021. 1. 29. · 2. Answer: (c) Explanation: Recently, a commemorative declaration marking the 75th anniversary of the signing of the Charter

`

26 | SEC-ECO-01

72. Answer: (b)

Explanation: Option (a) is correct: After the abolition of the Planning Commission of India, the NITI Aayog periodically estimates poverty lines and poverty ratios for each year. For this large Sample Surveys on Household Consumer Expenditure are conducted by the National Sample Survey Office (now National Statistics Organization) of the Ministry of Statistics and Programme Implementation. Normally these surveys are conducted every 5 years.

Option (b) is incorrect: In 2014, the government had scrapped the Rangarajan Committee report on poverty as it had pegged 100 million more poor vis-à-vis the last estimate based on the Tendulkar committee report. Option (c) is correct: In July 2013, based on the Tendulkar poverty line, the Planning Commission released poverty data for 2011-12. The number of poor in the country was pegged at 269.8 million

or 21.9% of the population. After this, no official poverty estimates in India have been released. Option (d) is correct: Suresh Tendulkar committee has moved away from calorie

intake as the criterion and considered per capita consumption expenditure on commodities and services. The Tendulkar committee steered from the calorie norm set in 1973-74—money required to access 2,100 calories in urban areas, and 2,400 calories in rural areas. It defines the new poverty line based on wider access to commodities and services like health, sanitation and education.

Extra Information: In 2015, NITI Aayog set up a Task Force on Poverty under the then Vice-Chairman, NITI Aayog, Prof Arvind Panagariya. The Task Force deliberated the issue of whether a Poverty Line is required. This panel backed the `Tendulkar poverty line’, which categorised people earning less than Rs. 33 a day as poor, on the ground that the line

is primarily meant to be an indicator for tracking progress in combating extreme poverty.

73. Answer: (d)

Explanation: In June 2020 the U.K. hosted the virtual Global Vaccine Summit. The summit raised almost £7bn to GAVI, the international vaccine alliance. India pledged 15 Million US Dollars. Statement 1 is incorrect: Global Alliance for Vaccines and Immunizations (GAVI) was created in 2000. It is an international organisation – a global Vaccine Alliance, bringing together public and private sectors with the shared goal of creating equal access to new and underused vaccines for children living in the world’s poorest countries. It is not created under the United Nations. Statement 2 is incorrect: GAVI does not conduct research and development for vaccine development. Instead, it funds scientific research for vaccine development. It supports a mission to save children’s lives and protect people’s health by increasing access to immunisation in poor countries. Its partners provide funding for vaccines and intellectual resources for care advancement. They contribute, also, to strengthening the capacity of the health system to deliver immunisation and other health services sustainably. Statement 3 is incorrect: GAVI is an independent organisation. It has its own secretariat office. It is not administered by the WHO.

Extra information: The members of this organization are developing countries and donor governments, the World Health Organization, UNICEF, the World Bank, the vaccine industry in both industrialised and developing countries, research and technical agencies, civil society, the Bill & Melinda Gates Foundation and other private philanthropists.

Page 28: BYJU’S IAS PRELIMS TEST SERIES 2021 · 2021. 1. 29. · 2. Answer: (c) Explanation: Recently, a commemorative declaration marking the 75th anniversary of the signing of the Charter

`

27 | SEC-ECO-01

74. Answer: (c)

Explanation: Statement 1 is correct: Inflation redistributes wealth in the society or from one category to another, such as from lenders to borrowers. Lenders lose by inflation because the money they get paid back has less buying power than the money they lent. Borrowers profit from inflation because the money they pay back is worth less than the money they borrowed, because the real value of money depreciated due to inflation. Statement 2 is incorrect: Rising inflation most of the time results into rising exchange rate owing to high fiscal and current account deficit. This increase may give a competitive edge to the exporters of goods and services, as the value of export decreases making exports cheaper resulting into higher sales. Statement 3 is correct: Inflation increases the nominal (face) value of wages, while their real value falls. That is why there is a negative impact of inflation on the purchasing power and living standard of wage employees. To neutralise this negative impact the Indian government provides a dearness allowance to its employees twice a year. Statement 4 is correct: Inflation increases employment in the short-run as per the Phillips curve, which states that rising inflation is a sign of increase in demand which in turn increases the demand of workers. However, the impact becomes neutral or even negative in the long run as the demand will become stagnant at a point and will decrease with rising inflation. Extra information: The Phillips curve shows that the higher the inflation rate, the lower the unemployment rate and vice versa. Thus, high levels of employment can only be reached at high inflation levels. Policies designed to promote economic growth, increase jobs and sustainable development

are highly dependent on the results of the Phillips curve. However, the results of the Phillips curve were found to be valid only in the short term. The Phillips curve does not explain circumstances of stagflation where both inflation and unemployment are alarmingly high.

75. Answer: (c)

Explanation: Statement 1 is correct: HDI uses Life Expectancy at birth as a measure of health, whereas HCI uses survival and stunting

rates at or below 5 years of age as a measure of health. Statement 2 is correct: HDI uses the number of years of schooling to measure education, whereas HCI uses quality-adjusted learning to measure education.

Statement 3 is incorrect: The United Nations Human Development Index is the benchmark which is the most widely used statistical measure of development across the world. The HDI is published by the United Nations Development Program (UNDP). The World Development Report, of which both the Human Capital Project and Human Capital Index are parts of, aims to measure the development levels of various countries using standardised parameters. Human Capital Index is published by the World Bank.

Extra Information:

The HCI measures outcomes instead of processes. As development is an ongoing process, measuring outcomes would be ineffective as the indicators are by nature very slow-moving, creating little incentive for countries to improve their HCI ranking.

The HCI is a number between 0 and 1 indicating the expected productivity of a child born now after they attain 18 years of age under the present conditions.

The HCI does not take into account per capita income levels into account while measuring development, which

Page 29: BYJU’S IAS PRELIMS TEST SERIES 2021 · 2021. 1. 29. · 2. Answer: (c) Explanation: Recently, a commemorative declaration marking the 75th anniversary of the signing of the Charter

`

28 | SEC-ECO-01

is a key indicator in all other indices quantifying development.

76. Answer: (c)

Explanation: A Small Finance Bank primarily conducts basic deposit acceptance and loan banking activities for unserved and underserved sections, including small business units, small and marginal farmers, micro and small enterprises, and non-organized sector entities. Statement 1 is correct: A Small Finance Bank is registered as a public limited company under the Companies Act, 2013. It is licensed and regulated under the Banking Regulation Act, 1949. Statement 2 is correct: A Small Finance Bank is required to extend 75% of its Adjusted Net Bank Credit (ANBC) to the priority sectors under priority sector lending (PSL) targets notified by the RBI. Although 40 per cent of its ANBC should be allocated to various PSL sub-sectors in accordance with existing PSL requirements, the bank can assign 35 percent of the balance to any one or more PSL sub-sectors in which it has a competitive advantage. Statement 3 is incorrect: According to the current Foreign Direct Investment (FDI) policy for private sector banks, foreign shareholding in Small Finance Banks is allowed. Statement 4 is correct: The Small Finance Bank cannot be a Business Correspondent (BC) for another bank. However, it can have its own BC network.

77. Answer: (d)

Explanation: Statement 1 is correct: The Producer Price Index (PPI) is a group of indices that calculates and represents the average change over time in the selling prices received by domestic producers for their output.

Statement 2 is incorrect: It measures the price changes from the producer’s perspective. The PPI is different from the Consumer Price Index in that it measures costs from the viewpoint of industries that make the products whereas the CPI measures prices from the perspective of consumers. As PPI measures the average price change received by the producer, it excludes indirect taxes in its calculation. Statement 3 is correct: While PPI tracks price change in both goods and services. Wholesale Price Index includes only goods but not services.

78. Answer: (d)

Explanation: Gross Value Added (GVA) can be defined as output produced after deducting the intermediate value of consumption. It is a measure of value-added in goods and services produced in the economy i. e. GVA = economic output – input. GVA is sector-specific while GDP is calculated by summation of GVA of all sectors of the economy with taxes added and subsidies deducted.

Statement 1 is incorrect: Gross Value Added (GVA) at basic prices includes production taxes, whereas it excludes

production subsidies available on the commodity. Statement 2 is incorrect: Gross Value Added (GVA) at factor cost includes all

subsidies and excludes all taxes. 79. Answer: (d)

Explanation: Socialism refers to the collective ownership of the means of production, central planning of the economy, and the emphasis on equality and economic security with the goal of reducing class distinctions. Statement 1 is correct: A socialist economy has cooperative collective farming. For example, cooperative farming followed under New Economic Policy, 1921 by the USSR.

Page 30: BYJU’S IAS PRELIMS TEST SERIES 2021 · 2021. 1. 29. · 2. Answer: (c) Explanation: Recently, a commemorative declaration marking the 75th anniversary of the signing of the Charter

`

29 | SEC-ECO-01

Statement 2 is incorrect: In a socialist society, the government decides what goods are to be produced in accordance with the needs of society. It is assumed that the government knows what is good for the people of the country and so the desires of individual consumers are not given much importance. Statement 3 is incorrect: The goods distribution on the basis of purchasing power is a basic element of the capitalist economy. Statement 4 is correct: Strictly, in a socialist society resources are owned by the state I.e., it is owned and controlled by the people collectively. Statement 5 is correct: A socialist nation provides free healthcare, education, and housing to all its citizens. In short, it is characterized by welfarism.

80. Answer: (b)

Explanation: Two central laboratories which are Pharmacopoeial Laboratory for Indian Medicine (PLIM) and Homoeopathic Pharmacopoeia Laboratory (HPL) were established as subordinate offices under the Ministry of Health and Family Welfare in 1970 and 1975 respectively. PLIM and HPL, both were designated as supporting structures of PCIM&H.

Statement 1 is incorrect: On 3rd June 2020, the erstwhile autonomous PCIM&H was re-established as a subordinate office under the Ministry of AYUSH by merging into it the two central laboratories, namely PLIM and HPL. Statement 2 is correct: Development of Pharmacopoeias and Formularies as well as acting as Central Drug Testing cum Appellate Laboratory for Indian systems of Medicine & Homoeopathy are the key fields of activity of PCIM&H.

81. Answer: (c)

Explanation: Fixed capital formation refers to a firm's

method of growing its fixed capital stock. Fixed capital is assets used in the production process that has been owned by a corporation for more than a year. (The creation of fixed capital does not cover existing raw materials used in the productive process). Fixed capital can also be referred to as Property, Plant, and Equipment (PP&E). For example, if a firm builds a new factory or invests in new machines, this will be an accumulation of fixed capital.

Statements 1 and 2 are incorrect: Examples of Fixed Capital Formations are:

Buying new building or expanding existing factory

Purchase of transport equipment

Office equipment, such as computers, printers

Machinery used in production. Statements 3 is correct: Currency and savings deposit in banks are not considered as fixed capital, it is liquid capital. Fixed capital is the assets used in the production process.

82. Answer: (b)

Explanation: Statement 1 is incorrect: CPI includes a basket of goods and services. It is not like the GDP deflator, which measures the economy with reference to all the goods in a country. Statement 2 is correct: CPI includes prices of goods and services consumed by the representative consumer, hence it includes prices of imported goods as well. Statement 3 is correct: CPI attempts to quantify the aggregate price level in an economy and thereby measures the purchasing power of a country's unit of currency.

Page 31: BYJU’S IAS PRELIMS TEST SERIES 2021 · 2021. 1. 29. · 2. Answer: (c) Explanation: Recently, a commemorative declaration marking the 75th anniversary of the signing of the Charter

`

30 | SEC-ECO-01

83. Answer: (c)

Explanation: Banks Board Bureau (BBB) is an autonomous advisory and recommendatory body created by the government to enhance the governance of the Public Sector Banks and state-owned financial institutions. The BBB was created after the recommendations of the Committee to Review Governance of Boards of Banks in India (PJ Nayak Committee, 2014). Statement 1 is incorrect: The BBB recommends candidates for the

appointment of the Board of Directors in PSBs and state-owned FIs (Whole-time Directors and Non-Executive Chairman). It has no power to appoint the directors and other members of the banks but its function is limited to the recommendation of suitable candidates. The final appointment is approved by the central

government. It also advises the Government on matters relating to appointments, confirmation or extension of tenure, and termination of services of the Board of Directors of the above-mentioned levels. Statement 2 is correct: BBB is also responsible for advising the government to evolve suitable business strategies and capital raising plans for the public sector banks. Statement 3 is incorrect: According to the PJ Nayak Committee recommendation, the BBB was also to be morphed into Bank Investment Company (BIC), which was envisaged as the holding company for PSBs. With the banking space seeing fast-paced changes vis-a-vis business models and technology, BIC would help in quick decisions without the fear of the 3Cs — Central Vigilance Commission, Comptroller and Auditor General, and Central Bureau of Investigation. The government has not yet accepted this recommendation.

Extra information: Some other function of BBB

To advise the Government on the desired structure at the Board level, and, for senior management personnel, for each PSB and Financial Institution (FI).

To help banks to develop a robust leadership succession plan for critical positions that would arise in the future through appropriate HR processes including performance management systems.

To build a data bank containing data relating to the performance of PSBs/FIs and their officers.

To advise the Government on the formulation and enforcement of a code of conduct and ethics for managerial personnel in PSBs/Financial Institutions.

To advise the Government on evolving suitable training and development programs for management personnel in PSBs/FIs.

84. Answer: (d)

Explanation: Statement 1 is incorrect: Certain goods, referred to as public goods (such as national defence, roads, government administration), as distinct from private goods (like clothes, cars, food items), cannot be provided through the market mechanism, i.e. by transactions between individual consumers and producers and must be provided by the government. This is the allocation function. Statement 2 is incorrect: Through its tax

and expenditure policy, the government attempts to bring about a distribution of income that is considered ‘fair’ by society. The government affects the personal disposable income of households by making transfer payments and collecting taxes and, therefore, can alter the income distribution. This is the distribution function.

Page 32: BYJU’S IAS PRELIMS TEST SERIES 2021 · 2021. 1. 29. · 2. Answer: (c) Explanation: Recently, a commemorative declaration marking the 75th anniversary of the signing of the Charter

`

31 | SEC-ECO-01

85. Answer: (b)

Explanation: The “Lost at Home” report has been published by the UN Children’s Fund (UNICEF).

Extra Information: Key findings of the report: 1. According to this report, around 12

million new displacements of children took place in 2019. In these, around 3.8 million of them were caused by conflict and violence, and the remaining 8.2 million, caused by disasters linked mostly to weather-related events.

2. Natural disasters were the cause for more large scale displacements than conflict and violence. Almost 10 million new displacements in 2019 were recorded in East Asia and the Pacific (39 %) and almost the same number in South Asia (9.5 million).

3. Intensifying suffering from coronavirus: Camps or informal settlements are often overcrowded and lack adequate health and hygiene services. There is often no physical distancing, creating conditions that are highly conducive to the spread of disease.

4. Risks internally displaced children face include child labour, child marriage, trafficking.

5. The largest number of internally displaced children due to conflict is found in the Middle East and North Africa (MENA) and sub-Saharan Africa. Internally displaced persons are concentrated in two regions — the Middle East and North Africa and West and Central Africa.

6. In India, more than five million people were internally displaced due to natural disasters, conflict, and violence in 2019, marking the world's largest number of new internal displacements during the period followed by the Philippines, Bangladesh, and China. India, the Philippines, Bangladesh, and China accounted for 69 percent of global displacements caused by disasters.

86. Answer: (a)

Explanation: Statement 1 is correct: Peer to Peer (P2P) lending is a method of crowdfunding used to raise unsecured loans and repaid with interest. Crowdfunding refers to supporting projects with small sums of money collected by a large number of individuals with the help of a portal (which acts as an intermediary). It uses an online platform that serves as a link between lenders and borrowers. Statement 2 is correct: The borrower may be either an individual or a legal person requiring a loan (say, an individual body, a HUF, a corporation, society, or some artificial body, whether incorporated or not). Statement 3 is incorrect: The interest rate is set on the electronic platform for every loan by means of a mutual agreement between the borrower and the lender or by the platform itself. The transfer of funds between participants on the P2P lending platform will take place via the mechanism of escrow accounts. All fund transfers shall be through and from bank accounts, and cash transactions are strictly prohibited.

87. Answer: (c)

Explanation: Statement 1 is correct: The real exchange rate measures prices abroad relative to those at home. Statement 2 is incorrect: The nominal exchange rate does not take inflation into account. It is the real exchange rate that takes inflation into account. Statement 3 is incorrect: Though largely exchange rates are market-determined, a central bank of a country can change the exchange rate by engaging in open market operations in the foreign exchange market. Thus by doing so, a country may gain an advantage in international trade if it controls the market for its currency to keep its value low. The International

Page 33: BYJU’S IAS PRELIMS TEST SERIES 2021 · 2021. 1. 29. · 2. Answer: (c) Explanation: Recently, a commemorative declaration marking the 75th anniversary of the signing of the Charter

`

32 | SEC-ECO-01

Monetary Fund aims to promote the stability of the international monetary and financial system and doesn’t play a direct role in fixing the exchange rates.

88. Answer: (c)

Explanation:

Recently in June 2020, US Congress passed the Uyghur Rights Bill. The Uyghurs are an ethnic and religious minority in China who mostly inhabit Xinjiang Province. Xinjiang is an autonomous region within China and is also its largest region. It is rich in minerals, and shares borders with seven countries – Mongolia, Russia, Kazakhstan, Kyrgyzstan, Tajikistan, Afghanistan, and India.

Source: https://www.voanews.com/

89. Answer: (a)

Explanation: Price elasticity of demand is defined as the percentage change in the quantity demanded of a good or service divided by the percentage change in the price. In other words, price elasticity of demand is the rate at which the demand increases or decreases with the corresponding price change. Statement 1 is correct: Price-elasticity of demand is a measure of the responsiveness of the demand for a good to changes in its price. Demands for some goods are very responsive to price changes while demands for certain others are not so responsive to price changes. Statement 2 is incorrect: The price elasticity of demand for the good depends on the nature of the good and the

availability of near substitutes for the good. For instance, essential food items are necessary for life, and the demand for such goods does not change much in response to changes in their prices. Food demand does not change much, even though food prices increase. On the other hand, demand for luxury goods can be very sensitive to price changes. In general, demand for life-essential goods is likely to be price inelastic, whereas demand for luxury products is likely to be price elastic.

90. Answer: (a)

Explanation: Statement 1 is correct: The MPI is an international measure of acute multidimensional poverty covering over 100 developing countries.

Figure: Weight distribution in the Multidimensional Poverty Index Source: UNDP

The MPI reflects both the incidence of multidimensional deprivation (a headcount of those in multidimensional poverty) and its intensity (the average deprivation score experienced by poor people). Statement 2 is incorrect: MPI assesses poverty at an individual level. MPI helps to identify the most vulnerable people – the poorest among the poor, revealing poverty patterns within countries, enabling policymakers to design policies more effectively. Statement 3 is incorrect: It was first developed in 2010 by Oxford Poverty and Human Development Initiative (OPHI) and the United Nations Development Programme (UNDP) for UNDP’s Human Development Reports.

Page 34: BYJU’S IAS PRELIMS TEST SERIES 2021 · 2021. 1. 29. · 2. Answer: (c) Explanation: Recently, a commemorative declaration marking the 75th anniversary of the signing of the Charter

`

33 | SEC-ECO-01

91. Answer: (c)

Explanation: Statement 1 is correct: Deposits held by Governments, banks and other institutions with the RBI are part of liabilities of the RBI. These represent the balances maintained with the Reserve Bank, by banks, the Central and State Governments (cash balances, MSS etc.), All India Financial Institutions, such as Export-Import Bank (EXIM Bank), NABARD, Foreign Central Banks, International Financial Institutions, balances in Employees’ Provident Fund, Depositor Education and Awareness Fund (DEA Fund), amount outstanding against Reverse Repo, Medical Assistance Fund (MAF) etc. Statement 2 is correct: The notes in circulation comprise:

The notes issued by the Government of India up to 1935 and by the RBI since then minus the notes held in the Banking Department, i.e. notes held outside Reserve Bank by the public, banks treasuries, etc.

Currency notes issued by the central bank as promissory notes. Therefore, currency notes constitute the liability of RBI.

Statement 3 is incorrect: India's Foreign Exchange Reserves comprise Foreign Currency Assets, Gold, SDR's and Reserve Bank positions with the International Monetary Fund (IMF). The foreign currency reserves act as assets of RBI. They can be used for multiple purposes like to back the issuance of notes along with rupee securities and gold. In the Banking Department, foreign currency reserves include foreign currency assets and balances with foreign entities like Bank for International Settlements (BIS), foreign commercial banks etc. Statement 4 is correct: Paid-up Capital and Reserve Fund: The Capital of the Bank is held by the Government of India and reserve funds i.e. Credit (Long-term Operations) Fund, National Agricultural Credit (Stabilisation) Fund, National

Industrial Credit (Long-term Operations) Fund of the Bank are part of other liability.

92. Answer: (c)

Explanation: It is a Central Sector Scheme to facilitate street vendors to access affordable working capital loan for resuming their livelihoods activities, after easing of lockdown. Statement 1 is incorrect: PM Street Vendor’s Atmanirbhar Nidhi (PM SVANidhi) is a special micro-credit facility for street vendors and not for farmers. It covers street vendors/hawkers vending in urban areas, as on or before March 24, 2020, including the vendors of surrounding peri-urban and rural areas. The Scheme shall be implemented up to March 2022. Statement 2 is incorrect: No collateral security is required to avail this loan. A beneficiary can avail loan of up to ₹ 10,000 at a subsidized rate of interest. This scheme also offers interest subsidy on timely or early repayment at 7%. It has a monthly cash-back incentive on digital transactions. It offers higher loan eligibility on timely repayment of the first loan. Statement 3 is correct: This scheme was launched by the Ministry of Housing and Urban Affairs.

Extra Information: Lending institutions that will provide credit under the scheme are Scheduled Commercial Banks, Regional Rural Banks, Small Finance Banks, Cooperative Banks, Non-BankingFinancial Companies, Micro-Finance Institutions and SHG Banks.

93. Answer: (a)

Explanation: In India, co-operative banks are registered under the States Cooperative Societies Act. They also come under the regulatory ambit of the Reserve Bank of India (RBI) under two laws, namely, the Banking

Page 35: BYJU’S IAS PRELIMS TEST SERIES 2021 · 2021. 1. 29. · 2. Answer: (c) Explanation: Recently, a commemorative declaration marking the 75th anniversary of the signing of the Charter

`

34 | SEC-ECO-01

Regulation Act, 1949, and the Banking Laws (Application to Co-operative Societies) Act, 1965. Statement 1 is correct: Co-operative banks are financial entities established on a co-operative basis and belong to their members. This means that the customers of a co-operative bank are also its owners. These banks provide a wide range of regular banking and financial services. In a cooperative bank, one shareholder has one vote whatever the number of shares he/she may hold. In a joint-stock bank, the voting right of a shareholder is determined by the number of shares he/she possesses. Statement 2 is incorrect: Co-operative banks are broadly classified into urban or rural co-operative banks based on their region of operation. Urban co-operative banks are classified into scheduled and non-scheduled banks.

94. Answer: (a)

Explanation: Statement 1 is correct: The Deposit Insurance and Credit Guarantee Corporation (DICGC) insures deposits of all commercial banks including branches of foreign banks functioning in India, local area banks, regional rural banks, all co-operative banks except Primary Cooperative Banks in India. Statement 2 is incorrect: The DICGC insures all deposits such as savings, fixed, current, recurring, etc. deposits except the following types of deposits:

Deposits of foreign Governments;

Deposits of Central/State Governments;

Inter-bank deposits;

Deposits of the State Land Development Banks with the State co-operative bank;

Any amount due on account of and deposit received outside India; and

Any amount, which has been specifically exempted by the corporation with the previous approval of the Reserve Bank of India.

Statement 3 is incorrect: The DICGC insures principal and interest up to a maximum amount of ₹ five lakhs. For example, if an individual had an account with a principal amount of 4,95,000 plus accrued interest of 4,000, the total amount insured by the DICGC would be 4,99,000. If, however, the principal amount in that account is five lakhs, the accrued interest would not be insured, not because it is an interest but because that is the amount over the insurance limit. Statement 4 is correct: Every deposit in different banks is insured separately. If you have deposits with more than one bank, the deposit insurance coverage limit is applied separately to the deposits in each bank.

95. Answer: (b)

Explanation: Statement 1 is correct:

Purchasing power parity (PPP) is a popular metric, used by macroeconomic analysts, that compares different countries' currencies through a "basket of goods" approach.

Purchasing power parity (PPP) allows for economists to compare economic productivity and standards of living between countries.

Some countries adjust their gross domestic product (GDP) figures to reflect PPP.

A basket of goods refers to a fixed set of consumer products and services whose price is evaluated on a regular basis, often monthly or annually. The goods in the basket are meant to be representative of the broader economy and are adjusted periodically to account for changes in consumer habits.

PPP is calculated based on the common basket of goods and services that the local currency can buy in another country.

Statement 2 is correct: The International Comparison Programme (ICP) is a worldwide statistical initiative to collect comparative price data and estimate the

Page 36: BYJU’S IAS PRELIMS TEST SERIES 2021 · 2021. 1. 29. · 2. Answer: (c) Explanation: Recently, a commemorative declaration marking the 75th anniversary of the signing of the Charter

`

35 | SEC-ECO-01

PPPs of the world's economies. Its main objective is to provide comparable international price and volume measures of GDP and expenditure aggregates among countries. Statement 3 is incorrect: India is the third largest country after China and the USA in PPP terms as per World Bank data. India accounts for 6.7% or $8,051 billion (out of the world's total of $119,547 billion) of global Gross Domestic Product (GDP) in terms of PPP as compared to 16.4 % in the case of China and 16.3 % for the US. These figures were released in June 2020, under the World Bank data for the reference year 2017, under the International Comparison Programme that adjusts for differences in the cost of living across economies of the World.

96. Answer: (c)

Explanation: In June 2020, Indian Gas Exchange, the first nationwide online delivery based gas trading platform was launched. Statements 1 is correct: Indian Gas Exchange (IGX) is the first nationwide online delivery based gas trading platform to ensure market-driven gas pricing. It is expected to lead to market-driven pricing and boost consumption by improving availability. IGX is fully automated with a web-based interface to provide a seamless trading experience to the customers and is powered by best-in-class technology from GMEX, one of the world’s leading digital exchange trading and post-trade technology providers. Statement 2 is incorrect: The IGX is a digital trading platform that will enable natural gas buyers and sellers to trade for imported natural gas in three hubs, Dahej and Hazira in Gujarat and Kakinada in Andhra Pradesh, both in the spot market and in the forward market. Imported liquified natural gas (LNG) will be regasified and sold to buyers via the exchange, removing the need for buyers and sellers to find each other. The price of domestically produced natural gas is

decided by the government. It will not be sold on the gas exchange. Statement 3 is correct: The IGX platform is incorporated by the Indian Energy Exchange (IEX) Ltd., India’s leading energy market platform. Incorporated as a wholly-owned subsidiary of the IEX, IGX will enable market participants to trade in standardized gas contracts.

Extra information: IGX will play an instrumental role in transforming India’s gas markets, positioning India as a sustainable economy, and enhancing the industry’s competitiveness. The competitive price discovery will facilitate the availability of gas at lower prices for the cross-spectrum of industries across India, stimulate demand, and facilitate greater investments in domestic gas exploration.

97. Answer: (d)

Explanation: Core Inflation shows a price rise in all goods and services excluding energy and food articles. Core inflation is the increase in the prices of goods and services but does not include those from the food and energy sectors. This measure of inflation excludes these items because their prices are much more volatile. It is most often calculated using the consumer price index (CPI), which is a measure of prices for a basket of goods and services.

98. Answer: (a)

Explanation: Statement 1 is correct: NITI Aayog develops mechanisms to formulate credible plans at the village level and aggregate these progressively at higher levels of government. Statement 2 is correct: It offers a platform for the resolution of inter-sectoral and inter-departmental issues in order to accelerate the implementation of the development agenda.

Page 37: BYJU’S IAS PRELIMS TEST SERIES 2021 · 2021. 1. 29. · 2. Answer: (c) Explanation: Recently, a commemorative declaration marking the 75th anniversary of the signing of the Charter

`

36 | SEC-ECO-01

Statement 3 is incorrect: Now scrapped Planning commission had the function, 'To prepare plans and allocate financial resources to such plans after consultation with the Finance Ministry.' But NITI Aayog doesn't have this function.

Extra information: Other functions of NITI Aayog are:

To focus on technology upgradation and capacity building for the implementation of programs and initiatives.

To evolve a shared vision of national development priorities sectors and strategies with the active involvement of States.

To foster cooperative federalism through structured support initiatives and mechanisms with the States.

To pay special attention to the sections of our society that may be at risk of not benefiting adequately from economic progress.

To provide advice and encourage partnerships between key stakeholders and think tanks, as well as educational and policy research institutions.

To create a knowledge, innovation and entrepreneurial support system through a collaborative community of experts, practitioners and partners.

99. Answer: (b)

Explanation: Statement 1 is incorrect: As per the Global TB Report 2019, the total number of TB cases notified during the last three years has increased in India.

Year Total No. of notified TB patients

2017 (Jan-Dec) 1827959

2018(Jan-Dec) 2155894

2019 (Jan-Dec) 2404815

Also, according to the Annual TB Report 2020: 1. 20.04 lakh notified TB patients in 2019

in India, which is a 14% increase over 2018 numbers.

2. Reduction in the number of missing cases to 2.9 lakh cases as against more than 10 lakh in 2017.

3. Private sector notifications increased by 35% with 6.78 lakh TB patients notified.

4. The proportion of children diagnosed with TB increased to 8% in 2019 compared to 6% in 2018.

Statements 2 is correct: The Annual TB Report is prepared and published by the Central TB Division, Ministry of Health and Family Welfare. Statement 3 is incorrect: The National Tuberculosis Elimination Program (NTEP) is a Centrally Sponsored Scheme being implemented under the aegis of the National Health Mission with resource sharing between the State Governments and the Central Government. The goal of the program is to achieve a TB-free India with zero deaths, disease and poverty due to tuberculosis.

100. Answer: (d)

Explanation: The rolling lockdowns due to COVID-19 have sent economies across the world into a tailspin. Growth in India, and indeed much of the world, is set to fall off a cliff in FY21. Economists have started to predict the shape of the economic recovery that can take place, once there is ease of lockdown restrictions. Statement 1 is correct: The most positive scenario in which the economy rapidly grows like a Phoenix after a recession is a Z-shaped recovery. Before settling back to the usual trend-line, it more than makes up for lost ground, thus creating a Z-shaped graph.

Page 38: BYJU’S IAS PRELIMS TEST SERIES 2021 · 2021. 1. 29. · 2. Answer: (c) Explanation: Recently, a commemorative declaration marking the 75th anniversary of the signing of the Charter

`

37 | SEC-ECO-01

Source: World Economic Forum

Statement 2 is correct: L shaped scenario exhibits a sharp decline in the economy, followed by a slow recovery period. It’s often punctuated by persistent unemployment, taking several years to recoup back to previous levels.

Source: World Economic Forum

Statement 3 is correct: The next-best scenario after the Z-shaped scenario is a V-shaped recovery in which the economy quickly recoups lost ground and gets back to the normal growth trend-line. A V-shaped recovery is characterized by a fast and sustained recovery after a sharp economic downturn in economic performance measurements. A V-shaped recovery is a best-case scenario given the recession, because of the pace of economic change and recovery in macroeconomic results.

Source: financialtimes.com

Page 39: BYJU’S IAS PRELIMS TEST SERIES 2021 · 2021. 1. 29. · 2. Answer: (c) Explanation: Recently, a commemorative declaration marking the 75th anniversary of the signing of the Charter

NASSCOM Design4India DesignAward 2018 for the ‘Best Design’

Mobile Category – 2018Express IT Awards for IT newsmaker

of the year 2017

Google DesignAward 2018

Business StandardStart-up of the year 2017

Google Play’s ‘Best Self Improvement’App in India – 2016

Deloitte Technology Fast 50 India and Fast 500 Asia Award Year2012, 2013, 2014, 2015, 2016,

2017

Education CompanyOf The Year 2016, 2018

VCCIRCLEAWARDS

Awards

byjus.com

To book a FREE COUNSELLING SESSION with our IAS Mentors call: 9241333666Visit https://byjus.com/ias/ for more details

Incredible results year after year!

20132014

20152016

2017

2018

2019

62 out of1228 vacancies

82 out of1364 vacancies

162 out of1164 vacancies

215 out of1209 vacancies

236 out of1058 vacancies

183 out of812 vacancies

165 out of829 vacancies

Selected candidates from BYJU'S

Ranks inTop 1004 Ranks in

Top 2009 Ranks inTop 501 3 Ranks in

Top 10022Congratulations to our toppers

RANK 03

Pratibha Verma

RANK 06

Vishakha Yadav

RANK 08

Abhishek Saraf

RANK 10

Sanjita Mohapatra